Exam 1 practice questions 320

Ace your homework & exams now with Quizwiz!

Upon evaluation of the patient's laboratory data and clinical signs and symptoms, the nurse suspects that the patient may have pheochromocytoma. Which of the following is directly related with pheochromocytoma? Select all that apply. 1. Severe headache; pain score of 9 out of 10 2. Perspiration 3. Blood pressure 80/90 mm Hg 4. Pallor 5. Lethargy

1, 2 Severe headache; pain score of 9 out of 10 Perspiration . Severe headache, perspiration, are indicative of pheochromocytoma. High, not low, blood pressure is strongly associated with pheochromocytoma. The massive release of catecholamines is associated with tremor, and nervousness, not lethargy. Palpitations may also be seen.

An 84-year-old client is experiencing severe arthritis pain. The nurse realizes that which of the following pain management approaches would be the most beneficial for this client? (Select all that apply.) 1. Avoid NSAIDs. 2. Utilize morphine or morphine-like medication 3. Provide medication through the oral route 4. Utilize diazepam 5. Suggest Darvocet 6. Provide medication through the intramuscular route

1, 2, 3 Avoid NSAIDs, utilize morphine or morphine-like medication, provide medication through the oral route When providing pain medication to a geriatric client, pain management approaches include the utilization of morphine or morphine-like medication to control pain and provide medication using the oral route. NSAIDs should also be avoided because of the risk of gastrointestinal bleeding. Diazepam should be avoided because of a long half-life. Darvocet should be avoided because of toxic effects with renal insufficiency. Medication should not be provided using the intramuscular route because of muscle wasting and loss of fatty tissue in the elderly client.

A 62-year-old woman is being seen at the health care provider's office for an initial evaluation of her symptoms of shortness of breath, cough, and sputum production. Her spirometry reveals an FEV1 of 65% predicted. Her FVC is 90%. Her FEV1/FVC is 60%. She has been given a diagnosis of COPD. She reports a smoking history of 35 years. She now smokes 10 cigarettes a day. She has tried to quit several times and was able to stay quit for 1 week a few years ago. She is concerned about weight gain if she quits smoking. The provider gave her a prescription for bupropion and nicotine nasal spray. The nurse is asked to review an approach to smoking cessation. What should be included in the teaching plan (Select all that apply) except: 1. Bupropion and nicotine replacement can assist in the quitting process 2. Successful smoking cessation includes multiple approaches 3. Dealing with the habit of smoking is important to success 4. Quitting smoking once COPD is diagnosed has no effect on prognosis

1, 2, 3 1. Bupropion and nicotine replacement can assist in the quitting process 2. Successful smoking cessation includes multiple approaches 3. Dealing with the habit of smoking is important to success Smoking cessation is a key factor for improving prognosis in COPD. Even patients with severe disease will benefit from smoking cessation. There is strong evidence that nicotine replacement and/or bupropion or nortriptyline increase long-term smoking abstinence. For patients concerned with weight gain during a quit attempt, bupropion may be helpful. However, bupropion cannot be given to patients with a history of seizures as it lowers the seizure threshold. Pharmacologic therapy is one aspect of a successful quit effort. Helping patients deal with their particular habits that trigger the urge to smoke should be included in the overall approach to smoking cessation

An elderly client being treated for type 2 diabetes mellitus begins to experience lethargy, weakness, and polyuria while recovering from cataract surgery. The nurse would suspect the client is developing hyperosmolar hyperglycemic nonketotic syndrome when which of the following is assessed? (Select all that apply.) 1. Blood glucose level 450 mg/dL 2. No ketones in the urine 3. Serum sodium 145 mEq/L 4. Serum osmolality 320 mOsm/kg 5. Blood pressure 120/68 mmHg 6. Heart rate 78 beats per minute

1, 2, 3, 4 Blood glucose level 450 mg/dl No ketones in the urine Serum sodium 145 mEq/L Serum osmolality 320 mOsm/kg Assessment findings consistent with hyperosmolar hyperglycemic nonketotic syndrome include a blood glucose level greater than 400 mg/dL, absence of ketones in the urine, serum sodium greater than 140 mEq/L, and serum osmolality greater than 310 mOsm/kg. The blood pressure of 120/68 mmHg is within normal limits. The heart rate of 78 beats per minute is within normal limits.

The circulating nurse is performing a time out prior to the beginning of a surgical procedure. Which of the following will be assessed during this time out? (Select all that apply.) 1. Correct client 2. Correct procedure 3. Correct site and side 4. Correct surgeon 5. Correct day 6. Correct time

1, 2, 3, 4 Correct client, procedure, site and side, surgeon A correctly performed time out includes verifying the right client; the correct procedure; the correct site and side; the correct surgeon; the correct position; the correct equipment, instruments, and implants if necessary. The correct day and time are not parts of the surgical time out.

A client diagnosed with chronic obstructive pulmonary disease is scheduled for diagnostic tests. Which of the following are used to aid in the diagnosis of this disorder? (Select all that apply.) 1. Pulmonary function spirometry tests 2. Chest x-ray 3. Electrocardiogram 4. Medication levels 5. Sputum samples 6. Electrolyte levels

1, 2, 3, 4 Pulmonary function spirometry tests, chest x-ray, electrocardiogram, medication levels Tests used to aid in the diagnosis of chronic obstructive pulmonary disease include pulmonary function spirometry tests, chest x-ray, electrocardiogram, and medication levels. Sputum samples are not useful and are not recommended in the diagnosis of chronic obstructive pulmonary disease. Electrolyte levels are not indicated.

When a client is brought from the surgical suite to the postanesthesia care unit, the nurse will conduct a rapid head-to-toe visual assessment. Which of the following statuses will be assessed during the initial assessment? (Select all that apply.) 1. Surgical site 2. Vital signs 3. Respiratory stability 4. Circulatory stability 5. Range of motion of lower extremities 6. Bowel sounds

1, 2, 3, 4 Surgical site, vital signs, respiratory stability, circulatory stability When a client is admitted to the postanesthesia care unit, the initial head-to-toe assessment includes surgical site, vital signs, respiratory stability, and circulatory stability. Range of motion of the lower extremities and bowel sounds are not a part of the initial head-to-toe assessment.

A client who has had abdominal surgery complains of feeling as though "something gave way" in the incisional site. The nurse removes the dressing and notes the presence of a loop of bowel protruding through the incision. Which interventions should the nurse take? Select all that apply. 1. Contact the surgeon. 2. Instruct the client to remain quiet. 3. Prepare the client for wound closure. 4. Document the findings and actions taken. 5. Place a sterile saline dressing and ice packs over the wound. 6. Place the client in a supine position without a pillow under the head.

1, 2, 3, 4 Contact the surgeon, instruct the client to remain quiet, prepare the client for wound closure, document the findings and actions taken Wound dehiscence is the separation of the wound edges. Wound evisceration is protrusion of the internal organs through an incision. If wound dehiscence or evisceration occurs, the nurse should call for help, stay with the client, and ask another nurse to contact the surgeon and obtain needed supplies to care for the client. The nurse places the client in a low-Fowler's position, and the client is kept quiet and instructed not to cough. Protruding organs are covered with a sterile saline dressing. Ice is not applied because of its vasoconstrictive effect. The treatment for evisceration is usually immediate wound closure under local or general anesthesia. The nurse also documents the findings and actions taken. Focus on the subject, that the client is experiencing wound evisceration. Visualizing this occurrence will assist you in determining that the client would not be placed supine and that ice packs would not be placed on the incision

The nurse is caring for a client diagnosed with pneumonia. Which of the following signs and symptoms would the nurse most likely assess in this client? (Select all that apply.) 1. Abdominal pain 2. Anorexia 3. Cough 4. Dyspnea 5. Fever 6. Frequent wiping of the nose

1, 2, 3, 4, 5 Abdominal pain, anorexia, cough, dyspnea, fever Specific symptoms suggestive of pneumonia include fever, chills or rigor, sweats, new cough (with or without sputum), pleuritic chest pain, and dyspnea. Nonspecific symptoms include malaise, fatigue, abdominal pain, headaches, anorexia, and worsening of an underlying illness. Frequent wiping of the nose is a sign of allergic rhinitis

The nurse is instructing a client diagnosed with type 2 diabetes mellitus on activities to reduce the onset of macrovascular complications. Which of the following should the nurse include in these instructions? (Select all that apply.) 1. Attain a normal body weight 2. Stop smoking 3. Increase activity 4. Keep blood pressure under control 5. Decrease fat intake 6. Ingest alcohol every day

1, 2, 3, 4, 5 Attain normal body weight, stop smoking, increase activity, keep blood pressure under control, decrease fat intake Macrovascular complications associated with type 2 diabetes mellitus can be controlled by addressing the modifiable risk factors. The risk factors include obesity, smoking, sedentary lifestyle, high blood pressure, and fat intake. This is what the nurse should include in the instructions to this client. The client should not be instructed to ingest alcohol every day.

The nurse suspects a client is experiencing the early signs of myxedema coma when which of the following is assessed? (Select all that apply.) 1. Reduced level of consciousness 2. Hypothermia 3. Hypoventilation 4. Hypotension 5. Bradycardia 6. Reduced urine output

1, 2, 3, 4, 5 Reduced level of consciousness, hypothermia, hypoventilation, hypotension, bradycardia Myxedema is a medical emergency. The client will present with a diminished level of consciousness, hypothermia, hypoventilation, hypotension, and bradycardia. Prior to the coma, the client may be depressed, confused, paranoid, or even manic. Reduced urine output is not associated with this disorder

Which of the following would indicate to the nurse that a client is experiencing an endocrine disorder that is affecting the neurological system? (Select all that apply.) 1. Tremors 2. Memory loss 3. Jitteriness 4. Nervousness 5. Loss of sensation in the feet 6. Nerve pain

1, 2, 3, 4, 5 Tremors, memory loss, jitteriness, nervousness, loss of sensation in the feet Common neurological findings with an endocrine disorder include tremors, memory loss, jitteriness, nervousness, and decreased sensation in the hands and feet. Nerve pain is not associated with an endocrine disorder affecting the neurological system

A client is receiving diagnostic tests to determine the presence of a malignant thyroid lesion. Which of the following are symptoms that are usually associated with a malignant thyroid? (Select all that apply.) 1. Hoarseness 2. Onset of dysphagia 3. Age 20; Male gender 4. Thyroid scan revealing a cold nodule 5. Soft nodules 6. Presence of a single firm nodule

1, 2, 3, 4, 6 Hoarseness, onset of dysphagia, age 20; male gender, thyroid scan revealing a cold nodule, presence of a single firm nodule Assessment findings consistent with a malignant thyroid lesion include hoarseness, dysphagia, young adult male; thyroid scan revealing a cold nodule; and the presence of a single firm nodule. Multiple soft nodules are indicative of benign thyroid lesions

The nurse determines that a client is experiencing a risk associated with the use of anesthesia for a surgical procedure. Which of the following are considered risks of anesthesia? (Select all that apply.) 1. Nausea and vomiting 2. Sore throat 3. Seizure 4. Postoperative myocardial infarction 5. Surgical wound infection 6. Hypothermia

1, 2, 3, 4, 6 Nausea and vomiting, sore throat, seizure, postoperative myocardial infarction, hypothermia Risks of anesthesia include adverse reaction to the anesthetic, nausea and vomiting, sore throat, seizure, myocardial infarction, hypothermia, malignant hyperthermia, numbness or loss of function of a body part, and disseminated intravascular coagulation. Surgical wound infection is not a risk associated with anesthesia.

The perioperative nurse is identifying nursing diagnoses appropriate for a client currently having surgery. Which of the following would be appropriate for the client at this time? 1. Risk for infection 2. Risk for impaired skin integrity 3. Risk for injury 4. Risk for inadequate nutrition 5. Risk for hypothermia 6. Risk for fluid volume overload

1, 2, 3, 5 Risk for infection, risk for impaired skin integrity, risk for injury, risk for hypothermia Nursing diagnoses for the perioperative client include risk for infection, risk for impaired skin integrity, risk of injury, and risk of hypothermia. Risk for inadequate nutrition and risk for fluid volume overload would be more appropriate during the postoperative period of client care.

Prior to hospitalization, a client had been ingesting high doses of oxycodone. The nurse suspects the client is experiencing symptoms of withdrawal when which of the following are assessed? (Select all that apply.) 1. Muscle twitching and spasms 2. Restlessness 3. Increased heart rate 4. Drop in blood pressure 5. Increase in blood pressure 6. Irritability

1, 2, 3, 5, 6 Muscle twitching and spasms, restlessness, increased heart rate, increase in blood pressure, irritability Withdrawal symptoms include myoclonus or muscle twitching and spasms, restlessness, irritability, increased heart rate, and increased blood pressure. A decrease in blood pressure is not a symptom of narcotic medication withdrawal.

Which of the strategies can a perioperative nurse use to make a child feel less anxious prior to a surgical procedure? (Select all that apply.) 1. Take the client on a tour of the operating room 2. Allow the client to bring a toy or stuffed animal 3. Allow the parents to stay with the child as much as possible 4. Have the chaplain say a prayer with the child 5. Use age-appropriate explanations 6. Respond to questions in a straightforward manner

1, 2, 3, 5, 6 Take the client on a tour of the operating room, allow the client to bring a toy or stuffed animal, allow the parents to stay with the child as much as possible, use age-appropriate explanations, respond to questions in a straightforward manner Strategies to help a preoperative pediatric client feel less anxious prior to a surgical procedure include taking the client on a tour of the operating room, allowing the client to bring a toy or stuffed animal, allowing the parents to stay with the client as much as possible, using age-appropriate explanations, and responding to questions in a straightforward manner. Having a chaplain say a prayer with the child is good, but it may not be age appropriate.

A client with severe pain from spinal stenosis is prescribed Methadone. The nurse realizes that the advantages of this medication are what? (Select all that apply.) 1. Decrease in the need for antidepressant adjuvant medication 2. Less frequent dosing schedule 3. Long half-life 4. Inexpensive 5. Can be used for intermittent pain 6. Does not cause respiratory depression

1, 2, 4 Decrease in the need for antidepressant adjuvant medication, less frequent dosing schedule, inexpensive The advantages of methadone include that it decreases the need for antidepressant adjuvant medication because it increases the release of serotonin and norepinephrine, dosing is every 12 hours, and it is inexpensive. Disadvantages of this medication include: it has a long half-life; it cannot be used for intermittent pain management; and it does cause respiratory depression.

A client diagnosed with severe arthritis tells the nurse that she always has some degree of pain. Which of the following could explain this clients poor pain management? (Select all that apply.) 1. Client does not appear to be in pain 2. Client does not report pain 3. Client cannot afford pain medication 4. Client is fearful of becoming addicted to pain medication 5. Client believes pain medication means the condition is worse 6. Client has a high pain tolerance

1, 2, 4, 5 Client does not appear to be in pain, client does not report pain, client is fearful of becoming addicted to pain medication, client believes pain medication means the condition is worse Barriers to pain assessment and management include that the client is not demonstrating overt signs of pain, and therefore she does not need pain medication; the client does not report pain, so therefore she does not need pain medication; the client is fearful of becoming addicted to pain medication; and the client believes pain medication means the condition is worse. The fact that the client is unable to afford pain medication and is having a high pain tolerance are not identified barriers to pain assessment and management.

A client has been diagnosed with hyperthyroidism. The nurse monitors for which signs and symptoms indicating a complication of this disorder? Select all that apply. 1. Fever 2. Nausea 3. Lethargy 4. Tremors 5. Confusion 6. Bradycardia

1, 2, 4, 5 Fever, nausea, tremors, confusion Thyroid storm is an acute and life-threatening complication that occurs in a client with uncontrollable hyperthyroidism. Signs and symptoms of thyroid storm include elevated temperature (fever), nausea, and tremors. In addition, as the condition progresses, the client becomes confused. The client is restless and anxious and experiences tachycardia Focus on the subject, signs and symptoms indicating a complication of hyperthyroidism. Recall that thyroid storm is a complication of hyperthyroidism. Options 3 and 6 can be eliminated if you remember that thyroid storm is caused by the release of thyroid hormones into the bloodstream, causing uncontrollable hyperthyroidism. Lethargy and bradycardia (think: slow down) are signs of hypothyroidism (slow metabolism).

Which of these instructions are for a client diagnosed with a pneumothorax? (Select all that apply.) 1. Remove air from the pleural space 2. Correct acid-base imbalances 3. Treat infection 4. Minimize damage 5. Reexpand the lung 6. Improve fluid balance

1, 2, 4, 5 Remove air from the pleural space, correct acid-base imbalances, minimize damage, re-expand the lung Treatment goals for pneumothorax include removing the air and fluid from the pleural space, correcting acid-base imbalance, minimizing further damage, and reexpanding the lung. Treating infection and improving fluid balance are not treatment goals for a pneumothorax.

A client is diagnosed with a thyroid storm. Which of the following will the nurse most likely assess in this client? (Select all that apply.) 1. Fever 2. Tachycardia 3. Hypotension 4. Restlessness 5. Cardiac arrhythmias 6. Sweating

1, 2, 4, 5, 6 Fever, tachycardia, restlessness, cardiac arrhythmias, sweating Clinical manifestations of a thyroid storm include fever, tachycardia, restlessness, cardiac arrhythmias, and sweating. Hypotension is not a clinical manifestation of a thyroid storm

The nurse would be concerned that a client is at risk for developing chronic pain when which of the following health problems are diagnosed? (Select all that apply.) 1. Osteoarthritis 2. Osteoporosis 3. Heart disease 4. Diabetes mellitus 5. Chronic pulmonary disease 6. Anemia

1, 2, 5 Osteoarthritis, osteoporosis, chronic pulmonary disease Common health problems associated with chronic pain include osteoarthritis, osteoporosis, and chronic pulmonary disease. Heart disease, diabetes mellitus, and anemia are not associated with chronic pain

The nurse is assessing a client diagnosed with type 2 diabetes mellitus for symptoms associated with diabetic ketoacidosis. Which of the following will the nurse most likely assess in this client? (Select all that apply.) 1. Dehydration 2. Fruity breath odor 3. Hypertension 4. Bradycardia 5. Kussmaul breathing 6. Abdominal pain

1, 2, 5, 6 Dehydration, fruity breath odor, Kussmaul breathing, abdominal pain The client diagnosed with diabetic ketoacidosis will experience dehydration, fruity breath odor, Kussmaul respirations, and abdominal pain. The client will also have hypotension and not hypertension. The clients heart rate will be tachycardic and not bradycardic.

The nurse is assessing the thorax of an elderly client. Which of the following would be considered normal age-related changes in this clients respiratory system? (Select all that apply.) 1. Hyperresonance 2. Pain with inspiration 3. Vital capacity reduced 4. Hemoptysis 5. Productive cough 6. Wheezes

1, 3 Hyperresonance, vital capacity reduced Normal age-related changes seen in the elderly include hyperresonance with palpation and a reduction in the vital capacity. Pain with inspiration, hemoptysis, productive cough, and wheezes are not normal age-related changes of the respiratory system.

The nurse is completing an assessment on a client who is being admitted for a diagnostic workup for primary hyperparathyroidism. Which client complaints would be characteristic of this disorder? Select all that apply. 1. Polyuria 2. Headache 3. Bone pain 4. Nervousness 5. Weight gain

1, 3 Polyuria, bone pain The role of parathyroid hormone (PTH) in the body is to maintain serum calcium homeostasis. In hyperparathyroidism, PTH levels are high, which causes bone resorption (calcium is pulled from the bones). Hypercalcemia occurs with hyperparathyroidism. Elevated serum calcium levels produce osmotic diuresis and thus polyuria. This diuresis leads to dehydration (weight loss rather than weight gain). Loss of calcium from the bones causes bone pain. Options 2, 4, and 5 are not associated with hyperparathyroidism. Some gastrointestinal symptoms include anorexia, nausea, vomiting, and constipation. Focus on the subject, assessment findings in hyperparathyroidism. Think about the pathophysiology associated with hyperparathyroidism. Remember that hypercalcemia is associated with this disorder and that hypercalcemia leads to diuresis, and that calcium loss from bone leads to bone pain.

The nurse is planning care for a client diagnosed with hypercalcemia caused by hyperparathyroidism. Which of the following should the nurse add as interventions to this clients care plan? (Select all that apply.) 1. Administer high volume intravenous fluids as prescribed 2. Monitor arterial blood gases 3. Calculate sodium chloride intake to achieve 400 mEq each day 4. Provide low rates of intravenous fluids 5. Provide thyroid replacement medication orally 6. Monitor body temperature

1, 3 Administer high volume intravenous fluids as prescribed, calculate sodium chloride intake to achieve 400 mEq each day Management of fluid and electrolytes is the priority for a client diagnosed with hypercalcemia caused by hyperparathyroidism. The client needs intensive hydration with intravenous normal saline. The nurse also needs to ensure that the client receives greater than 400 mEq of sodium chloride each day. The other answer choices are interventions appropriate for a client diagnosed with myxedema.

A client has been diagnosed with allergic rhinitis. Which of the following should the nurse instruct the client regarding strategies to avoid this disorder? (Select all that apply.) 1. Remove home carpeting 2. Reduce the use of an air conditioner 3. Remove pets from the home 4. Open windows in the spring and summer 5. Use feather pillows 6. Wash bed linens in cold water

1, 3 Remove home carpeting Remove pets from the home Strategies to reduce the symptoms of allergic rhinitis include removing home carpeting and removing pets from the home. The client should be instructed to use an air conditioner, keep windows closed during allergy season, avoid feather pillows, and wash bed linens in hot water.

A client with a diagnosis of Addisonian crisis is being admitted to the intensive care unit. Which findings will the interprofessional health care team focus on? Select all that apply. 1. Hypotension 2. Leukocytosis 3. Hyperkalemia 4. Hypercalcemia 5. Hypernatremia

1, 3 Hypotension, Hyperkalemia In Addison's disease, also known as adrenal insufficiency, destruction of the adrenal gland leads to decreased production of adrenocortical hormones, including the glucocorticoid cortisol and the mineralocorticoid aldosterone. Addisonian crisis, also known as acute adrenal insufficiency, occurs when there is extreme physical or emotional stress and lack of sufficient adrenocortical hormones to manage the stressor. Addisonian crisis is a life-threatening emergency. One of the roles of endogenous cortisol is to enhance vascular tone and vascular response to the catecholamines epinephrine and norepinephrine. Hypotension occurs when vascular tone is decreased and blood vessels cannot respond to epinephrine and norepinephrine. The role of aldosterone in the body is to support the blood pressure by holding salt and water and excreting potassium. When there is insufficient aldosterone, salt and water are lost and potassium builds up; this leads to hypotension from decreased vascular volume, hyponatremia, and hyperkalemia. The remaining options are not associated with Addisonian crisis. Focus on the subject, Addisonian crisis. Think about the pathophysiology associated with Addison's disease. Recalling that in Addison's disease there is a decrease in the glucocorticoid cortisol and the mineralocorticoid aldosterone will assist in determining the correct answer.

During morning rounds on a medical floor, the nurse assesses a patient admitted the previous evening with an exacerbation of COPD. The patient is complaining of increased dyspnea. He is sitting on the side of the bed with his arms braced against his knees. His respiratory rate was 20 on admission and is now 28. His breath sounds are diminished with coarse crackles heard throughout. His oxygen saturation is 93% on 2 L/min of supplemental oxygen. The nurse would perform which of the following (Select all that apply) except: 1. Administer albuterol using a spacer that has been ordered "as needed" 2. Switch the patient's oxygen to a venturi-mask at 32% 3. Instruct the patient to begin pursed-lip breathing 4. Instruct the patient in controlled cough

1, 3, 4 Administer albuterol using a spacer that has been ordered "as needed" Instruct the patient to begin pursed-lip breathing Instruct the patient in controlled coughing . Albuterol is a sympathomimetic bronchodilator (short acting beta adrenergic or SABA) used to relieve bronchospasm and improve secretion clearance. Pursed-lip breathing and controlled cough may further help the patient's dyspnea and secretion clearance. The nurse should monitor the effect of treatment. Worsening or unrelieved symptoms and/or worsening oxygen saturation will need further evaluation and treatment. The patient's oxygen saturation is adequate and there is no need to increase the FiO2 (fracture of inspired oxygen) at this time.

The nurse is teaching a client how to use a nasal spray. Which of the following should be included in these instructions? (Select all that apply.) 1. Blow the nose before instilling the spray 2. Tilt the head back and angle the tip of the bottle to the side of the nostril 3. Use a finger to occlude the nostril that is not receiving the spray 4. Inhale gently and evenly while discharging the spray into the nostril 5. If a second spray is recommended, immediately repeat the procedure 6. Blow the nose after administration of the spray

1, 3, 4 Blow the nose before instilling the spray Use a finger to occlude the nostril that is not receiving the spray Inhale gently and evenly while discharging the spray into the nostril For the steps to be correct, the head should be slightly forward, the second spray should be given 15 to 20 seconds after the spray, and the client should not blow the nose after the administration of the spray. The client should be instructed to blow the nose before instilling the spray, to use a finger to occlude the nostril that is not receiving the spray, and to gently inhale while the spray is being delivered into the nostril.

The nurse is monitoring a client who was diagnosed with type 1 diabetes mellitus and is being treated with NPH and regular insulin. Which manifestations would alert the nurse to the presence of a possible hypoglycemic reaction? Select all that apply. 1. Tremors 2. Anorexia 3. Irritability 4. Nervousness 5. Hot, dry skin 6. Muscle cramps

1, 3, 4 Decreased blood glucose levels produce autonomic nervous system symptoms, which are manifested classically as nervousness, irritability, and tremors. Option 5 is more likely to occur with hyperglycemia. Options 2 and 6 are unrelated to the manifestations of hypoglycemia. In hypoglycemia, usually the client feels hunger. Focus on the subject, a hypoglycemic reaction. Think about the pathophysiology and manifestations that occur when the blood glucose is low. Recalling the signs of this type of reaction will direct you easily to the correct options

The nursing management of a patient who underwent transsphenoidal removal of a pituitary tumor yesterday includes which of the following actions? Select all that apply. 1. Maintaining oral care 2. Removing nasal pack to check for bleeding and CSF leak 3. Giving fluid after nausea, and then slowly progressing to normal diet 4. Raising the head of the bed to promote drainage 5. Brush teeth to prevent bacterial overgrowth with hard toothbrush

1, 3, 4 Maintaining oral care, giving fluid after nausea and then slowly progressing to normal diet, raising the head of the bed to promote drainage Nasal packs are not removed until the third or fourth postoperative day. Removing the nasal pack the day after surgery may exacerbate bleeding. If a sublabial approach is used, the patient is advised not to brush his or her teeth until the incision above the teeth has been healed.

The nurse is preparing a list of home care instructions for a client who has been hospitalized and treated for tuberculosis. Which instructions should the nurse include on the list? Select all that apply. 1. Activities should be resumed gradually. 2. Avoid contact with other individuals, except family members, for at least 6 months. 3. A sputum culture is needed every 2 to 4 weeks once medication therapy is initiated. 4. Respiratory isolation is not necessary, because family members already have been exposed. 5. Cover the mouth and nose when coughing or sneezing and put used tissues in plastic bags. 6. When 1 sputum culture is negative, the client is no longer considered infectious and usually can return to former employment.

1, 3, 4, 5 Activities should be resumed gradually, a sputum culture is needed every 2 to 4 weeks once medication therapy is initiated, respiratory isolation is not necessary because family members already have been exposed, cover the mouth and nose when coughing or sneezing and put used tissues in plastic bags The nurse should provide the client and family with information about tuberculosis and allay concerns about the contagious aspect of the infection. The client needs to follow the medication regimen exactly as prescribed and always have a supply of the medication on hand. Side and adverse effects of the medication and ways of minimizing them to ensure compliance should be explained. After 2 to 3 weeks of medication therapy, it is unlikely that the client will infect anyone. Activities should be resumed gradually and a well-balanced diet that is rich in iron, protein, and vitamin C to promote healing and prevent recurrence of infection should be consumed. Respiratory isolation is not necessary, because family members already have been exposed. Instruct the client about thorough hand washing, to cover the mouth and nose when coughing or sneezing, and to put used tissues into plastic bags. A sputum culture is needed every 2 to 4 weeks once medication therapy is initiated. When the results of 3 sputum cultures are negative, the client is no longer considered infectious and can usually return to former employment. Focus on the subject, home care instructions for tuberculosis. Knowledge regarding the pathophysiology, transmission, and treatment of tuberculosis is needed to answer this question. Read each option carefully to answer correctly.

The community health nurse is conducting an educational session with community members regarding the signs and symptoms associated with tuberculosis. The nurse informs the participants that tuberculosis is considered as a diagnosis if which signs and symptoms are present? Select all that apply. 1. Dyspnea 2. Headache 3. Night sweats 4. A bloody, productive cough 5. A cough with the expectoration of mucoid sputum

1, 3, 4, 5 Dyspnea, night sweats, a bloody productive cough, a cough with the expectoration of mucoid sputum Tuberculosis should be considered for any clients with a persistent cough, weight loss, anorexia, night sweats, hemoptysis, shortness of breath, fever, or chills. The client's previous exposure to tuberculosis should also be assessed and correlated with the clinical manifestations. Note the subject, clinical manifestations of tuberculosis. Note that headache is not specifically associated with tuberculosis, is not respiratory in nature, and is not associated with an infection to assist in eliminating this option.

A client is demonstrating signs of chronic sinusitis. Which of the following will the nurse most likely assess in this client? (Select all that apply.) 1. Facial pain 2. Fever 3. Headache 4. Toothache 5. Fatigue 6. Swollen neck glands

1, 3, 4, 5 Facial pain, headache, toothache, fatigue Manifestations of chronic sinusitis include facial pain, headache, toothache, and fatigue. Fever and swollen neck glands would indicate the disorder has spread beyond the sinuses

A client is scheduled for a ventilation-perfusion scan. The nurse realizes that this diagnostic test is used to diagnose which of the following? (Select all that apply.) 1. Pulmonary emboli 2. Congestive heart failure 3. Bronchitis 4. Asthma 5. Pneumonia 6. COPD

1, 3, 4, 5, 6 Pulmonary emboli, bronchitis, asthma, pneumonia, COPD The purpose of the ventilation-perfusion scan is to diagnose and locate pulmonary emboli. It is also helpful in diagnosing bronchitis, asthma, pneumonia, COPD, and cancer. This scan is not used to diagnose congestive heart failure.

With which of the following can the nurse instruct a client who is experiencing pain from a sore throat? (Select all that apply.) 1. Gargle with warm salt water 2. Eat salty foods 3. Suck on hard candy 4. Drink fluids 5. Avoid citrus fruits 6. Suck on popsicles

1, 3, 4, 6 Gargle with warm salt water, suck on hard candy, drink fluids, suck on popsicles Interventions to reduce the pain from a sore throat include gargling with warm salt water, sucking on throat lozenges or hard candy, sucking on flavored frozen desserts or popsicles, using a humidifier in the bedroom, and drinking fluids. The client should not be instructed to eat salty foods or avoid citrus fruits

The nurse is documenting that a client has adventitious breath sounds. Which of the following would be considered this type of sound? (Select all that apply.) 1. Rales 2. Vesciular 3. Rhonchi 4. Wheeze 5. Bronchovesicular 6. Pleural friction rub

1, 3, 4, 6 Rales, Rhonchi, Wheeze, Pleural friction rub Adventitious breath sounds include rales, rhonchi, wheezes, and pleural friction rubs. Vesicular and bronchovesicular are considered normal breath sounds.

The nurse is planning to administer the pneumococcus vaccination to a client. Which of the following would indicate that a client is a candidate for this vaccination? (Select all that apply.) 1. Age 70 2. Age 55 3. Diagnosis of heart failure 4. Recovering from knee placement surgery 5. Diagnosis of asthma 6. Recovering from an appendectomy

1, 3, 5 Age 70, diagnosis of heart failure, diagnosis of asthma Criteria for the pneumococcus vaccination include high-risk groups such as people over age 65, diagnosed with chronic heart disease, and diagnosed with asthma. Age 55, recovering from knee replacement surgery; and recovering from an appendectomy are not criteria for the pneumococcus vaccination.

The nurse is admitting a client who is diagnosed with syndrome of inappropriate antidiuretic hormone secretion (SIADH) and has serum sodium of 118 mEq/L (118 mmol/L). Which primary health care provider prescriptions should the nurse anticipate receiving? Select all that apply. 1. Initiate an infusion of 3% NaCl. 2. Administer intravenous furosemide. 3. Restrict fluids to 800 mL over 24 hours. 4. Elevate the head of the bed to high-Fowler's. 5. Administer a vasopressin antagonist as prescribed.

1, 3, 5 Intiate an infusion of 3% NaCl Restrict fluids to 800 mL over 24 hours Administer a vasopressin antagonist as prescribed Clients with SIADH experience excess secretion of antidiuretic hormone (ADH), which leads to excess intravascular volume, a declining serum osmolarity, and dilutional hyponatremia. Management is directed at correcting the hyponatremia and preventing cerebral edema. Hypertonic saline is prescribed when the hyponatremia is severe, less than 120 mEq/L (120 mmol/L). An intravenous (IV) infusion of 3% saline is hypertonic. Hypertonic saline must be infused slowly as prescribed, and an infusion pump must be used. Fluid restriction is a useful strategy aimed at correcting dilutional hyponatremia. Vasopressin is an ADH; vasopressin antagonists are used to treat SIADH. Furosemide may be used to treat extravascular volume and dilutional hyponatremia in SIADH, but it is only safe to use if the serum sodium is at least 125 mEq/L (125 mmol/L). When furosemide is used, potassium supplementation should also occur and serum potassium levels should be monitored. To promote venous return, the head of the bed should not be raised more than 10 degrees for the client with SIADH. Maximizing venous return helps avoid stimulating stretch receptors in the heart that signal to the pituitary that more ADH is needed Focus on the subject, treatment for SIADH. Think about the pathophysiology associated with SIADH. Remember that SIADH is associated with the increased secretion of ADH, or vasopressin. Excess vasopressin leads to increased intravascular fluid volume, decreased serum osmolality, and hyponatremia. When hyponatremia and decreased serum osmolality become severe, cerebral edema occurs.

The nurse is caring for a client diagnosed with cystic fibrosis. Which of the following medications does the nurse realize are commonly used to help treat this disorder? (Select all that apply.) 1. N-acetylcysteine (Mucomyst) 2. Acetaminophen (Tylenol) 3. Dornase alfa (Pulmozyme) 4. Furosemide (Lasix) 5. Ibuprofen (Motrin) 6. Digitalis (Digoxin)

1, 3, 5 N-acetylcysteine, Dornase alfa, Ibuprofen Medications commonly used to treat cystic fibrosis include N-acetylcysteine (Mucomyst), Dornase alfa (Pulmozyme), and Ibuprofen (Motrin). Acetaminophen, furosemide, and digitalis are not routinely prescribed in the treatment of cystic fibrosis.

The nurse determines that a client is experiencing chronic pain when which of the following is assessed? (Select all that apply.) 1. Suffering 2. Fatigue 3. Sleeplessness 4. Apathy 5. Sadness 6. Anger

1, 3, 5 Suffering, sleeplessness, sadness The descriptor triad for chronic pain is suffering, sleeplessness, and sadness. Fatigue, apathy, and anger do not describe chronic pain.

Which of the following symptoms would suggest to the nurse that a client is experiencing symptoms of pheochromocytoma? (Select all that apply.) 1. Severe headache 2. Decreased urine output 3. Palpitations 4. Diarrhea 5. Profuse sweating 6. Weight gain

1, 3, 5 Severe headache, palpitations, profuse sweating Severe headache, palpitations, and profuse sweating are the most common symptoms of pheochromocytoma. Decreased urine output, diarrhea, and weight gain are not associated with this disorder

The nurse is assessing a client for decreased fremitus. Which of the following conditions are associated with decreased fremitus? (Select all that apply.) 1. Atelectasis 2. Emphysema 3. Pneumonia 4. Pneumothorax 5. Pulmonary fibrosis 6. Pulmonary infarction

1, 3, 5, 6 Atelectasis, Pneumonia, Pulmonary fibrosis, Pulmonary infarction Atelectasis, pneumonia, pulmonary fibrosis, and pulmonary infarction cause decreased fremitus. Pneumothorax and emphysema would cause increased fremitus

The nurse is using the PAINAID Scale to assess a clients level of pain. Which of the following are assessed with this pain scale? (Select all that apply.) 1. Breathing rate 2. Assign a number to the degree of pain 3. Negative vocalizations 4. Assign a facial expression to the degree of pain 5. Facial expression 6. Body language

1, 3, 5, 6 Breathing rate, negative vocalizations, facial expression, body language The PAINAID scale assesses breathing, negative vocalizations, facial expression, body language, and comfort. The Numerical Rating Scale assigns a number to the degree of pain. The WongBaker FACES Scale assigns a facial expression to the degree of pain

The nurse, planning care for a client diagnosed with a pneumothorax, identifies which types of pneumothorax? (Select all that apply.) 1. Spontaneous 2. Radical 3. Traumatic 4. Incomplete 5. Iatrogenic 6. Tension

1, 3, 5, 6 Spontaneous, traumatic, Iatrogenic, tension The four types of pneumothorax are spontaneous, traumatic, iatrogenic, and tension. Radical and incomplete are not types of pneumothorax

The nurse is planning care for a client diagnosed with bronchiolectasis. Which of the following would be goals for this clients care? (Select all that apply.) 1. Treat the infection 2. Reduce the heart rate 3. Minimize further damage 4. Improve urine output 5. Promote breathing 6. Remove secretions

1, 3, 5, 6 Treat the infection, minimize further damage, promote breathing, remove secretions Treatment goals for the client diagnosed with bronchiolectasis include treat the infection, minimize further damage, promote effective airway breathing, and remove secretions. Treatment goals do not include reducing heart rate and improving urine output

A patient presented to the unit with an ADH-secreting tumor. Upon diagnostic and physical evaluation, the nurse suspects the patient is experiencing SIADH. Which of the following is a clinical manifestation of SIADH? Select all that apply. 1.Hyponatremia 2. Hypernatremia 3. Increased serum osmolality 4. Reduced serum osmolality 5. Dry mucous membranes 6. Low urine output

1, 4,6 Hyponatremia, Reduced serum osmolality, low urine output The patient with SIADH has hyponatremia (plasma sodium concentration of less than 135 mEq/L) and is not hypernatremia. Unlike healthy people, patients with SIADH cannot excrete diluted urine. As a result, they retain water (thus the serum osmolality decreases) and hyponatremia is seen

A client is prescribed a bedside diagnostic test to assess pulmonary status. The nurse will prepare to administer which of the following to the client? (Select all that apply.) 1. Capnography 2. Thoracentesis 3. Oximetry 4. Bronchoscopy 5. Polysomnography 6. Lung biopsy

1,3,5 Capnography, Oximetry, Polysomnography The three diagnostic tests that can be administered at the bedside include capnography, which measures exhaled carbon dioxide; oximetry, which measures oxygenation; and polysomnography, which measures breathing while asleep. Thoracentesis, bronchoscopy, and lung biopsy are all invasive procedures and cannot be administered at the bedside

The home health nurse visits a client with a diagnosis of type 1 diabetes mellitus. The client reports a history of vomiting and diarrhea and tells the nurse that no food has been consumed for the last 24 hours. Which additional statement by the client indicates a need for further teaching? 1. "I need to stop my insulin." 2. "I need to increase my fluid intake." 3. "I need to monitor my blood glucose every 3 to 4 hours." 4. "I need to call my primary health care provider (PHCP) because of these symptoms."

1. "I need to stop my insulin" When a client with diabetes mellitus is unable to eat normally because of illness, the client still should take the prescribed insulin or oral medication. The client should consume additional fluids and should notify the PHCP. The client should monitor the blood glucose level every 3 to 4 hours. The client should also monitor the urine for ketones during illness. Note the strategic words, need for further teaching. These words indicate a negative event query and the need to select the incorrect statement. Remembering that the client needs to take insulin will direct you easily to the correct option.

The nurse is teaching a client with hyperparathyroidism how to manage the condition at home. Which response by the client indicates the need for additional teaching? 1. "I should consume less than 1 liter of fluid per day." 2. "I should use my treadmill or go for walks daily." 3. "I should follow a moderate-calcium, high-fiber diet." 4. "My alendronate helps keep calcium from coming out of my bones."

1. "I should consume less than 1 liter of fluid per day" In hyperparathyroidism, clients experience excess parathyroid hormone (PTH) secretion. A role of PTH in the body is to maintain serum calcium homeostasis. When PTH levels are high, there is excess bone resorption (calcium is pulled from the bones). In clients with elevated serum calcium levels, there is a risk of nephrolithiasis. One to two liters of fluids daily should be encouraged to protect the kidneys and decrease the risk of nephrolithiasis. Moderate physical activity, particularly weight-bearing activity, minimizes bone resorption and helps protect against pathological fracture. Walking, as an exercise, should be encouraged in the client with hyperparathyroidism. Even though serum calcium is already high, clients should follow a moderate-calcium diet, because a low-calcium diet will surge PTH. Calcium causes constipation, so a diet high in fiber is recommended. Alendronate is a bisphosphate that inhibits bone resorption. In bone resorption, bone is broken down and calcium is deposited into the serum. Note the strategic words, need for additional teaching. These words indicate a negative event query and the need to select the incorrect statement. Consider the pathophysiology of hyperparathyroidism. Hyperparathyroidism leads to bone demineralization, which places the client at risk for pathological fracture, and high serum calcium, which places the client at risk for nephrolithiasis. Knowing that fluids should be encouraged rather than limited to help prevent nephrolithiasis should direct you to the correct option

The nurse is teaching a client about coughing and deep-breathing techniques to prevent postoperative complications. Which statement is most appropriate for the nurse to make to the client at this time as it relates to these techniques? 1. "Use of an incentive spirometer will help prevent pneumonia." 2. "Close monitoring of your oxygen saturation will detect hypoxemia." 3. "Administration of intravenous fluids will prevent or treat fluid imbalance." 4. "Early ambulation and administration of blood thinners will prevent pulmonary embolism."

1. "Use of an incentive spirometer will help prevent pneumonia" Postoperative respiratory problems are atelectasis, pneumonia, and pulmonary emboli. Pneumonia is the inflammation of lung tissue that causes productive cough, dyspnea, and lung crackles and can be caused by retained pulmonary secretions. Use of an incentive spirometer helps prevent pneumonia and atelectasis. Hypoxemia is an inadequate concentration of oxygen in arterial blood. While close monitoring of the oxygen saturation will help detect hypoxemia, monitoring is not directly related to coughing and deep-breathing techniques. Fluid imbalance can be a deficit or excess related to fluid loss or overload, and surgical clients are often given intravenous fluids to prevent a deficit; however, this is not related to coughing and deep breathing. Pulmonary embolus occurs as a result of a blockage of the pulmonary artery that disrupts blood flow to 1 or more lobes of the lung; this is usually due to clot formation. Early ambulation and administration of blood thinners helps prevent this complication; however, it is not related to coughing and deep-breathing techniques. Note the strategic words, most appropriate. Focus on the subject, client instructions related to coughing and deep-breathing techniques. Also, focus on the data in the question and note the relationship between the words coughing and deep-breathing in the question and pneumonia in the correct option.

An adult client is complaining of vision changes and difficulty speaking because the tongue is larger. The client also states that his shoes no longer fit. Based on these symptoms, the client is most likely to be diagnosed with: 1. Acromegaly 2. Cretinism 3. Gigantism 4. Graves disease

1. Acromegaly Acromegaly is caused by a hypersecretion of the pituitary growth hormone over a long period. This hypersecretion causes a coarsening of the features, including soft tissue overgrowth such as the tongue. Shoes and rings may no longer fit due to tissue and bone overgrowth. In children, hypersecretion of growth hormone causes gigantism. Cretinism and Graves disease are caused by a thyroid hormone imbalance

A 45-year-old woman with a diagnosis of asthma comes to the emergency room with worsening symptoms of cough and wheeze. She has been using her albuterol inhaler every 3 hours for the past 2 days. She denies allergies to medications, foods, and the environment. She has no other major medical problems and has never been a smoker. She is recovering from a viral respiratory infection. Her respiratory rate is 28; she is sitting upright on the stretcher. Her breath sounds are markedly diminished. Her room air oxygen saturation is 89%. The nurse should take the following next step: 1. Administer oxygen to keep her oxygen saturation at 90-92% or above 2. Recommend discharge with follow-up to her primary care provider (PCP) 3. Administer inhaled corticosteroids 4. Review asthma triggers and early warning signs

1. Administer oxygen to keep her oxygen saturation at 90-92% or above This patient is having a severe exacerbation of asthma with worsening oxygen saturation that has not responded to initial treatment (status asthmaticus). Oxygen therapy is appropriate in addition to close monitoring, inhaled sympathomimetics and systemic corticosteroids. Heliox may also be useful. She may need admission to the hospital for continued treatment and observation. Additionally, she needs to be monitored for signs of impending respiratory failure. Any patient admitted with asthma becomes high risk for worsening disease. While it is important to review asthma triggers and early warning signs, this should be done once the patient is stabilized

The nurse is caring for a client diagnosed with chronic obstructive pulmonary disease. Which of the following interventions require extra care by the nurse? 1. Administering pain medications 2. Applying a cardiac monitor 3. Encouraging fluids 4. Teaching the client diaphragmatic breathing

1. Administering pain medications Administering pain medications (narcotics) requires extra care by the nurse because these medications can depress respiratory status and worsen hypercapnia. Increasing fluids helps thin the clients secretions and is encouraged. Applying a cardiac monitor and monitoring the rhythm is part of a normal assessment. Teaching diaphragmatic breathing does not require extra care.

A client is diagnosed with primary adrenal insufficiency. The nurse realizes that this disorder affects which of the following glands? 1. Adrenal cortex 2. Adrenal medulla 3. Thyroid 4. Pituitary

1. Adrenal cortex Mineralocorticoids, glucocorticoids, and androgens are produced in the adrenal cortex. The principal mineralocorticoid is aldosterone. The adrenal medulla secretes the catecholamines. The thyroid and pituitary do not secrete aldosterone.

A client is diagnosed with benign cysts on the cortex of the adrenal glands. Which of the following hormones will be affected with this health problem? 1. Aldosterone and cortisol 2. Calcitonin and parathyroid hormone 3. Epinephrine and norepinephrine 4. Prolactin and luteinizing hormone

1. Aldosterone and cortisol Aldosterone and cortisol are released by the adrenal cortex. The adrenal medulla releases epinephrine and norepinephrine. Calcitonin and parathyroid hormone are released by the thyroid and parathyroid, respectively. Prolactin and luteinizing hormone are anterior pituitary hormones.

Which of the following should the nurse do when caring for an elderly postoperative client? 1. Allow rest periods between activities 2. Address the client by the first name 3. Assess for confusion if the client takes a long time to complete a task 4. Avoid eye contact

1. Allow rest periods between activities Caring for an elderly postoperative client, the nurse should allow rest periods between activities, avoid using the clients first name, not mistake slow activity for confusion, and maintain eye contact and full attention

A client is diagnosed with an increased level of glucagon. The nurse realizes that the production of glucagon occurs in which of the following cells within the pancreatic islets of Langerhans? 1. Alpha 2. Beta 3. Delta 4. F cells

1. Alpha Alpha cells produce and secrete glucagon. Beta cells produce and secrete insulin. Delta and F cells are responsible for somatostatin and pancreatic polypeptide, respectively.

A client is informed that a tricyclic antidepressant medication is going to help control his chronic pain. The nurse would expect the physician to prescribe: 1. Amitriptyline 2. Baclofen 3. Gabapentin 4. Diazepam

1. Amitriptyline Amitriptyline is an antidepressant. Gabapentin is an anticonvulsant. Baclofen is a muscle relaxant. Diazepam is a benzodiazepine.

The nurse is caring for a client diagnosed with acute sinusitis. Which of the following symptoms is the client most likely experiencing? 1. Anosmia 2. Fever 3. Halitosis 4. Metallic taste

1. Anosmia Clients often complain of unilateral face pain, purulent nasal discharge, pain during mastication, anosmia (absence of smell), and headache. Less common symptoms include fever, nasal congestion, halitosis, toothache, metallic taste, and cough.

The nurse is identifying diagnoses appropriate for a client scheduled for a surgical procedure. Which of the following is a diagnosis commonly used for preoperative client? 1. Anxiety 2. Sleep deprivation 3. Excess fluid volume 4. Disturbed body image

1. Anxiety The preoperative experience may be one of the most tension-producing periods of hospitalization. The nursing diagnosis anxiety is commonly used for preoperative clients. The other diagnoses are not commonly used as preoperative diagnoses.

The nurse receives a telephone call from the postanesthesia care unit stating that a client is being transferred to the surgical unit. The nurse plans to take which action first on arrival of the client? 1. Assess the patency of the airway. 2. Check tubes or drains for patency. 3. Check the dressing to assess for bleeding. 4. Assess the vital signs to compare with preoperative measurements.

1. Assess the patency of the airway The first action of the nurse is to assess the patency of the airway and respiratory function. If the airway is not patent, the nurse must take immediate measures for the survival of the client. The nurse then takes vital signs followed by checking the dressing and the tubes or drains. The other nursing actions should be performed after a patent airway has been established. The first action of the nurse is to assess the patency of the airway and respiratory function. If the airway is not patent, the nurse must take immediate measures for the survival of the client. The nurse then takes vital signs followed by checking the dressing and the tubes or drains. The other nursing actions should be performed after a patent airway has been established.

A nurse is caring for a postoperative patient on POD 2. The patient had a large upper abdominal incision. While assessing the patient at the beginning of the shift, the nurse noted decreased breath sounds, crackles, and a mild cough. What is the patient most likely experiencing? 1. Atelectasis 2. Pneumonia 3. Acute bronchitis 4. Hypoxemia

1. Atelectasis Many factors such as age, smoking, long duration of surgery, incisional site (chest and upper abdomen), as well as comorbidities and associated with an increased risk for postoperative pulmonary complications. Atelectasis remains a risk for the patient who is not moving well, not coughing well or performing deep-breathing as a result of a large incisional site. The nurse must be alert and aware of these signs and symptoms.

The nurse is assessing a client diagnosed with emphysema. Which of the following will most likely be assessed during the clients physical examination? 1. Barrel chest 2. Pectus carinatum 3. Pectus excavatum 4. Scoliosis

1. Barrel chest Barrel chest is often seen in chronic emphysema as a result of long-term air trapping. Pectus carinatum is an abnormal protuberance of the sternum, and pectus excavatum is an abnormal depression of the sternum

A client is demonstrating a crescendo-decrescendo pattern of breathing with periods of apnea. The nurse would document this breathing pattern as being: 1. Cheyne-Stokes 2. apnea 3. bradypnea 4. Kussmaul

1. Cheyne-Stokes Cheyne-Stokes breathing is a pattern of crescendo-decrescendo breathing. Apnea is the absence of breathing. Bradypnea is a breathing rate of less than 12 respirations per minute. Kussmaul breathing is rapid and deep and often associated with diabetic ketoacidosis.

A client diagnosed with a hemothorax has had a chest tube inserted and attached to a portable water-seal drainage system. Which of the following interventions would be inappropriate for this client? 1. Clamp the tubing when ambulating 2. Date and mark the amount of drainage in the collection chamber every shift 3. Monitor the suction chamber for continuous bubbling 4. Watch the water-seal chamber for fluctuation

1. Clamp the tubing when ambulating The chest tube should not be clamped or raised above the chest when ambulating. All other options are appropriate

A client tells the nurse that he is so thirsty that he has already consumed four pitchers of water. The clients urine output is 3500 mL in an 8-hour period. The client is recovering from surgery on the pituitary gland. What endocrine disorder is the client most likely experiencing? 1. Diabetes insipidus 2. Diabetes mellitus 3. Myxedema 4. Syndrome of inappropriate antidiuretic hormone secretion

1. Diabetes insipidus Diabetes insipidus and diabetes mellitus both cause increased urine output, but diabetes insipidus is related to a problem with antidiuretic hormone; diabetes mellitus is a problem with glucose. Myxedema is caused by a thyroid hormone imbalance. Syndrome of inappropriate antidiuretic hormone secretion causes fluid retention.

A client is demonstrating signs of respiratory alkalosis. The nurse realizes that this alteration is least likely caused by which of the following? 1. Diarrhea 2. Fever 3. Pain 4. Severe anemia

1. Diarrhea Diarrhea is a cause of metabolic acidosis. Causes of the respiratory alkalosis are hypoxia, increased minute ventilation, hyperventilation, pregnancy, fever, pain, and severe anemia.

A client with acquired immunodeficiency syndrome (AIDS) has histoplasmosis. The nurse should assess the client for which expected finding? 1. Dyspnea 2. Headache 3. Weight gain 4. Hypothermia

1. Dyspnea Histoplasmosis is an opportunistic fungal infection that can occur in the client with AIDS. The infection begins as a respiratory infection and can progress to disseminated infection. Typical signs and symptoms include fever, dyspnea, cough, and weight loss. Enlargement of the client's lymph nodes, liver, and spleen may occur as well. Focus on the subject, manifestations of histoplasmosis. Recalling that histoplasmosis is an infectious process will help you eliminate option 4. Because the client has AIDS and another infection, weight gain is an unlikely symptom and can be eliminated next. Knowing that histoplasmosis begins as a respiratory infection helps you choose dyspnea over headache as the correct option.

A client is scheduled for surgery in 2 weeks. Which of the following should the nurse instruct the client regarding healthy lifestyle behaviors? 1. Eat nutritious meals 2. If obese, cut calories before the surgery 3. If sedentary, exercise more before the surgery 4. Stop all prescribed medications

1. Eat nutritious meals The client should be encouraged to adopt healthy dietary, rest, and exercise habits before the surgery. A client who has not followed healthy lifestyle habits should not suddenly make these changes before a surgical procedure. The nurse should encourage the client to eat nutritious meals. A client who is obese should not be encouraged to cut calories before the surgery. The client who is sedentary should not be encouraged to suddenly exercise before the surgery. The client should not be instructed to stop prescribed medications unless a physician has prescribed this action

A female client is admitted with hyperprolactinemia. Which of the following would not be a clinical manifestation of the disorder in this client? 1. Excessive estrogen 2. Hirsutism 3. Osteoporosis 4. Weight gain

1. Excessive estrogen Hyperprolactinemia is associated with a decrease in estrogen, resulting in symptoms of vaginal dryness, hot flashes, osteopenia, and osteoporosis. The patient may also experience weight gain, irritability, hirsutism, anxiety, and depression.

A client who has undergone preadmission testing has had blood drawn for serum laboratory studies, including a complete blood count, coagulation studies, and electrolytes and creatinine levels. Which laboratory result should be reported to the surgeon's office by the nurse, knowing that it could cause surgery to be postponed? 1. Hemoglobin, 8.0 g/dL (80 mmol/L) 2. Sodium, 145 mEq/L (145 mmol/L) 3. Serum creatinine, 0.8 mg/dL (70.6 mcmol/L) 4. Platelets, 210,000 cells/mm3 (210 × 109/L)

1. Hemoglobin, 8/0 g/dL (80 mmol/L) Routine screening tests include a complete blood count, serum electrolyte analysis, coagulation studies, and a serum creatinine test. The complete blood count includes the hemoglobin analysis. All of these values are within normal range except for hemoglobin. If a client has a low hemoglobin level, the surgery likely could be postponed by the surgeon Focus on the subject, an abnormal laboratory result that needs to be reported. Use knowledge of the normal reference intervals to assist in answering correctly. The hemoglobin value is the only abnormal laboratory finding.

A client has a central nervous system disorder. The nurse realizes that the client may be experiencing alterations in hormones regulated by which of the following organs?' 1. Hypothalamus 2. Pineal gland 3. Pituitary gland 4. Thyroid

1. Hypothalamus The hypothalamus is considered the major regulating organ of the body because it is the connection between the nervous system and the endocrine system. The other organs take direction from the hypothalamus through the central nervous system.

A client has been instructed regarding a prolactin level to be drawn the next day. Which of the following statements indicate that the client will need further instruction? 1. I will be on time, in the afternoon 2. I will be relaxed 3. I will make sure not to take my antihistamine 4. I will practice another method of birth control rather than the pill

1. I will be on time, in the afternoon Certain medications (e.g., antihistamines and oral contraceptives) and fear can increase the prolactin level. The prolactin level is drawn in the morning

The nurse would expect that insulin may be substituted for other antidiabetic agents in which of the following patients? 1. In a patient with a history of DM hospitalized for an acute infection 2. In a patient having difficulty with weight management 3. In a patient experiencing hypoglycemia 4. In a patient who is newly diagnosed with borderline hyperglycemia

1. In a patient with a history of DM hospitalized for an acute infection Illness and infection increase blood sugar levels, making diabetes difficult to control with oral antidiabetic agents alone.

The nurse is monitoring the status of a postoperative client in the immediate postoperative period. The nurse would become most concerned with which sign that could indicate an evolving complication? 1. Increasing restlessness 2. A pulse of 86 beats per minute 3. Blood pressure of 110/70 mm Hg 4. Hypoactive bowel sounds in all 4 quadrants

1. Increasing restlessness Increasing restlessness is a sign that requires continuous and close monitoring because it could indicate a potential complication such as hemorrhage, shock, or pulmonary embolism. A blood pressure of 110/70 mm Hg with a pulse of 86 beats per minute is within normal limits. Hypoactive bowel sounds heard in all 4 quadrants are a normal occurrence in the immediate postoperative period. Note the strategic word, most. Focus on the subject, a manifestation of an evolving complication in the immediate postoperative period. Eliminate each of the incorrect options because they are comparable or alike and are normal expected findings, especially given the time frame noted in the question.

A client is being treated for exacerbation of chronic obstructive pulmonary disease. Which of the following nursing interventions will the nurse expect to be completed? 1. Initiate oxygen at 1 L/min via nasal cannula 2. Limit fluids 3. Place on respiratory isolation 4. Schedule all activities at one time

1. Initiate oxygen at 1 L/min via nasal cannula Oxygen for a client diagnosed with COPD should be low flow so as not to diminish the clients drive to breath. Fluids are encouraged, and activities should be interspersed with rest periods so the client will not become overtired. Isolation is not necessary at this time.

The nurse is taking the history of a client with occupational lung disease (silicosis). The nurse should assess whether the client wears which item during periods of exposure to silica particles? 1. Mask 2. Gown 3. Gloves 4. Eye protection

1. Mask Silicosis results from chronic, excessive inhalation of particles of free crystalline silica dust. The client should wear a mask to limit inhalation of this substance, which can cause restrictive lung disease after years of exposure. Options 2, 3, and 4 are not necessary. Focus on the subject, prevention of silicosis. Recalling that exposure to silica dust causes the illness and that the dust is inhaled into the respiratory tract will direct you to the correct option.

Which of the following should the nurse instruct a client who is recovering from a thyroid biopsy as an outpatient? 1. Notify the physician of any problems with breathing after the procedure 2. Go to the emergency room with any signs of a sore throat after the procedure 3. Expected to be admitted to the hospital if the surgeon decides to remove the thyroid after the biopsy 4. Perform no special preparations for the test

1. Notify the physician of any problems with breathing after the procedure The possibility of hematoma formation and edema post-procedure are the major complications that may present as respiratory difficulty. The client should be instructed to notify the physician of any problems with breathing after the procedure. A sore throat after a thyroid biopsy is a common experience, and the client does not need to go to the emergency room. The surgeon will not remove the thyroid gland during the same time as the biopsy. There are special preparations for the test, depending upon the level of anesthesia the client will receive.

The following day the patient's symptoms are improving. He is less dyspneic and able to walk to the bathroom with minimal assistance. In preparing for discharge, a home oxygen assessment is ordered. The patient's room air oxygen saturation is 88% at rest and 86% with exertion. Oxygen administered by nasal cannula at 2 L/min raises the oxygen saturation to 94% at rest and 92% when walking. Home oxygen is ordered as 2 L/min 24 hours a day. Teaching about home oxygen would include: 1. Oxygen improves cardiac and cognitive function and improves prognosis 2. Oxygen needs to be worn only when the patient is short of breath 3. Oxygen improves cardiac and cognitive function but does not improve prognosis 4. Home oxygen is dangerous and will explode

1. Oxygen improves cardiac and cognitive function and improves prognosis Oxygen has been shown by key studies to improve vital organ function and improve prognosis. Symptoms are an unreliable method of determining the need for oxygen. Many individuals may be dyspneic and have normal/near-normal oxygen saturation. Conversely, other individuals may deny symptoms, but be profoundly hypoxemic. Measuring oxygen levels by arterial blood gas or oximetry needs to be done to determine the need for supplemental oxygen. Oxygen supports combustion but it can be used safely in the home environment. Patients need to be taught to keep their oxygen away from open flame to decrease the risk of uncontrolled fire.

The nurse is monitoring a client newly diagnosed with diabetes mellitus for signs of complications. Which sign or symptom, if frequently exhibited in the client, indicates that the client is at risk for chronic complications of diabetes if the blood glucose is not adequately managed? 1. Polyuria 2. Diaphoresis 3. Pedal edema 4. Decreased respiratory rate

1. Polyuria Chronic hyperglycemia, resulting from poor glycemic control, contributes to the microvascular and macrovascular complications of diabetes mellitus. Classic symptoms of hyperglycemia include polydipsia, polyuria, and polyphagia. Diaphoresis may occur in hypoglycemia. Hypoglycemia is an acute complication of diabetes mellitus; however, it does not predispose a client to the chronic complications of diabetes mellitus. Therefore, option 2 can be eliminated because this finding is characteristic of hypoglycemia. Options 3 and 4 are not associated with diabetes mellitus. Focus on the subject, chronic complications of diabetes mellitus. Recall that poor glycemic control contributes to development of the chronic complications of diabetes mellitus. Remember the 3 Ps associated with hyperglycemia—polyuria, polydipsia, and polyphagia.

A client who is human immunodeficiency virus (HIV)-positive has had a tuberculin skin test (TST). The nurse notes a 7-mm area of induration at the site of the skin test and interprets the result as which finding? 1. Positive 2. Negative 3. Inconclusive 4. Need for repeat testing

1. Positive The client with HIV infection is considered to have positive results on tuberculin skin testing with an area of induration larger than 5 mm. The client without HIV is positive with an induration larger than 10 mm. The client with HIV is immunosuppressed, making a smaller area of induration positive for this type of client. It is possible for the client infected with HIV to have false-negative readings because of the immunosuppression factor. Options 2, 3, and 4 are incorrect interpretations. Eliminate options 3 and 4 first because they are comparable or alike. From the remaining options, recalling that the client with HIV infection is immunosuppressed will assist in determining the interpretation of the area of induration.

Blood work of a female client shows an increase in the production of estradiol. The nurse realizes that this hormone is controlled by: 1. Positive feedback 2. Negative feedback 3. Nervous feedback 4. Reverse feedback

1. Positive feedback Even though most of the hormones in the endocrine system are under a negative feedback mechanism, estradiol is not one of those hormones. Estradiol is controlled by a positive feedback mechanism in that when it increases, there will be in an increase in the production of folliclestimulating hormone by the anterior pituitary.

The nurse is reviewing a surgeon's prescription sheet for a preoperative client that states that the client must be nothing by mouth (NPO) after midnight. The nurse should call the surgeon to clarify that which medication should be given to the client and not withheld? 1. Prednisone 2. Ferrous sulfate 3. Cyclobenzaprine 4. Conjugated estrogen

1. Prednisone Prednisone is a corticosteroid. With prolonged use, corticosteroids cause adrenal atrophy, which reduces the ability of the body to withstand stress. When stress is severe, corticosteroids are essential to life. Before and during surgery, dosages may be increased temporarily and may be given parenterally rather than orally. Ferrous sulfate is an oral iron preparation used to treat iron deficiency anemia. Cyclobenzaprine is a skeletal muscle relaxant. Conjugated estrogen is an estrogen used for hormone replacement therapy in postmenopausal women. These last 3 medications may be withheld before surgery without undue effects on the client. Focus on the subject, the medication that should be administered in the preoperative period. Use knowledge about medications that may have special implications for the surgical client. Prednisone is a corticosteroid. Recall that when stress is severe, such as with surgery, corticosteroids are essential to life.

The nurse is planning care for the client diagnosed with viral rhinitis. Which of the following would be the best goal of care for this client? 1. Prevent secondary bacterial infection 2. Prevent rhinitis medicamentosa 3. Refrain from use of analgesics 4. Encourage complete participation in activities

1. Prevent secondary bacterial infection Treatment of acute rhinitis, or the common cold, is aimed at decreasing the impact of the symptoms and preventing secondary bacterial infection. Rhinitis medicamentosa occurs from misuse of nasal decongestants. Acetaminophen or a nonsteroidal anti-inflammatory agent is useful for fever, aches, and pain. Rest is encouraged.

What instruction should the nurse stress when teaching the preoperative patient about correct use of the PCA device? 1. Push the button when you feel the pain beginning rather than waiting until the pain is at its worst. 2. Push the button every 15 minutes whether you feel pain at that time or not. 3. Instruct your family or visitors to press the button for you when you are sleeping. 4. Try to go as long as you possibly can before you press the button.

1. Push the button when you feel the pain beginning rather than waiting until the pain is at its worst A patient experiencing pain can administer small amounts of medication directly into the IV, subcutaneous, or epidural catheter by pressing a button. The pump then delivers a preset amount of medication. The patient should be instructed to administer medication before the pain becomes severe

The nurse is assessing a client diagnosed with asthma. The clients breath sounds initially had wheezing but are diminishing until no audible sounds are heard. This has occurred because: 1. Swelling has increased, and it has blocked airways 2. The attack has passed 3. The client used an inhaler 4. No mucus is present

1. Swelling has increased, and it has blocked airways This client needs to be evaluated immediately and receive prompt treatment to reduce the airway obstruction and reverse inflammation. Lack of audible breath sounds does not mean that the attack has passed, the client has used an inhaler, or there is no mucus present.

The nurse is instructing a client on ways to reduce the transmission of tuberculosis. Which of the following should be included in these instructions? 1. The disease is transmitted by inhaling droplets exhaled by an infected person 2. The disease is transmitted by not fully cooking foods 3. The disease is transmitted by not washing hands 4. The disease is transmitted by sexual contact

1. The disease is transmitted by inhaling droplets exhaled by an infected person Tuberculosis is transmitted by inhaling the bacillus present in the air. The bacillus is present in the air after an infected person has coughed, sneezed, or expectorated. Tuberculosis is not transmitted through poorly cooked foods, poor handwashing, or sexual contact.

The nurse has just reassessed the condition of a postoperative client who was admitted 1 hour ago to the surgical unit. The nurse plans to monitor which parameter most carefully during the next hour? 1. Urinary output of 20 mL/hr 2. Temperature of 37.6° C (99.6° F) 3. Blood pressure of 100/70 mm Hg 4. Serous drainage on the surgical dressing

1. Urinary output of 20 mL/hr Urine output should be maintained at a minimum of 30 mL/hr for an adult. An output of less than 30 mL for 2 consecutive hours should be reported to the surgeon. A temperature higher than 37.7° C (100° F) or lower than 36.1° C (97° F) and a falling systolic blood pressure, lower than 90 mm Hg, are usually considered reportable immediately. The client's preoperative or baseline blood pressure is used to make informed postoperative comparisons. Moderate or light serous drainage from the surgical site is considered normal. Note the strategic word, most. Focus on the subject, expected postoperative assessment findings. To answer this question correctly, you must know the normal ranges for temperature, blood pressure, urinary output, and wound drainage. Note that the urinary output is the only observation that is not within the normal range

Which of the following can the nurse do to help an elderly client scheduled for a surgical procedure? 1. Work at a slower pace 2. Speed up the pace so the client has time to rest 3. Talk to family members and leave the client alone 4. Send them to the surgical holding area in advance

1. Work at a slower pace When caring for elderly clients, pace is important. Nurses should slow the pace. The nurse should not ignore the client. The nurse should also not send the client to the surgical holding area in advance since this could prove to be uncomfortable for the elderly client.

A client complains that the bed sheets touching his skin are extremely painful. The nurse realizes this client is experiencing: 1. allodynia 2. modulation 3. kinesthesia 4. proprioception

1. allodynia Allodynia or hyperalgesia is a state where a slight or nonpainful stimulus is interpreted as very painful. Kinesthesia is the awareness of movement. Proprioception is the awareness of body position. Modulation is an influencing factor in the perception of pain.

A client receiving around-the-clock medication for terminal cancer experiences additional pain when performing activities of daily living. The nurse realizes this client is experiencing: 1. breakthrough pain 2. intractable pain 3. psychosomatic pain 4. acute pain

1. breakthrough pain Breakthrough pain is commonly seen in the advanced stages of cancer. It is spontaneous, unpredictable, and can be initiated by certain activities such as during activities of daily living. Intractable pain is resistant to some or all forms of therapy. Psychosomatic pain is that which has a psychological origin. The client is diagnosed with terminal cancer. Acute pain has a sudden onset and resolves within 6 months.

The nurse, caring for a postoperative client, will assess vital signs: 1. every 15 minutes for the first hour 2. every 20 minutes for the first hour 3. every 30 minutes for the first hour 4. not important at this point

1. every 15 minutes for the first hour Vital signs are performed every 15 minutes for the first hour and may be done more often if the client is less stable. Vital sign assessment is extremely important and should be done more frequently than every 20 or 30 minutes

A client tells the nurse that she rarely experiences pain, but when she does, she seeks medical attention. The nurse realizes this client understands that pain is important because it: 1. is a protective system 2. includes the automatic withdrawal reflex 3. creates sensitivity to pain 4. helps with healing

1. is a protective system Pain is a protective system that includes protection from unsafe behaviors by use of reflexes, memory, and avoidance. Even though the automatic withdrawal reflex is a part of the pain response, it does not explain why pain is important. Pain does not create sensitivity to pain. Pain does not help with healing

A client being prepared for surgery has a pulse oximeter placed on one digit of his hand. The nurse is applying this device to monitor the clients: 1. oxygen level 2. heart rate 3. blood pressure 4. urine output

1. oxygen level Pulse oximeters are used to precisely identify the clients peripheral tissue oxygenation. Pulse oximeters are not to measure heart rate, blood pressure, or urine output.

A client is scheduled for x-rays of the long bones. The nurse realizes this diagnostic test is useful to help diagnose a disorder of the: 1. pituitary gland 2. pancreas 3. thyroid gland 4. adrenal gland

1. pituitary gland X-rays of the long bones is used to help diagnose disorders of the pituitary gland. This test is not used to diagnose disorders of the pancreas, thyroid, or adrenal glands.

A client is experiencing a sudden onset of headache, nausea, cough, fever, myalgia, and fatigue. The nurse suspects this client is experiencing: 1. seasonal influenza 2. chronic obstructive pulmonary disease 3. asthma 4. cystic fibrosis

1. seasonal influenza Seasonal influenza has a sudden onset with a headache, nausea, cough, chills, fever, rhinitis, myalgia, and extreme fatigue. These symptoms are not seen in chronic obstructive pulmonary disease, asthma, or cystic fibrosis.

The nurse is caring for a client hospitalized with acute exacerbation of chronic obstructive pulmonary disease. Which findings would the nurse expect to note on assessment of this client? Select all that apply. 1. A low arterial PCo2 level 2. A hyperinflated chest noted on the chest x-ray 3. Decreased oxygen saturation with mild exercise 4. A widened diaphragm noted on the chest x-ray 5. Pulmonary function tests that demonstrate increased vital capacity

2, 3 A hyperinflated chest noted on the chest x-ray Decreased oxygen saturation with mild exercise Clinical manifestations of chronic obstructive pulmonary disease (COPD) include hypoxemia, hypercapnia, dyspnea on exertion and at rest, oxygen desaturation with exercise, and the use of accessory muscles of respiration. Chest x-rays reveal a hyperinflated chest and a flattened diaphragm if the disease is advanced. Pulmonary function tests will demonstrate decreased vital capacity. Focus on the subject, manifestations of COPD. Think about the pathophysiology associated with this disorder. Remember that hypercapnia, a hyperinflated chest, a flat diaphragm, oxygen desaturation on exercise, and decreased vital capacity are manifestations.

A 55-year-old female presents to the clinic with complaints of fatigue and tiredness. The nurse notices that the patient's skin is thin, fragile, and easily traumatized. Ecchymosis and purple striae are noted over the thighs and abdomen. She presents with a slight kyphosis and a protruding abdomen. Which of the following methods of management might be appropriate for her? 1. Increase dose of corticosteroids 2. Unilateral or bilateral adrenalectomy 3. Increase dose of Spironolactone 4. Diet that is high in carbohydrates and low in protein

2, 3 Unilateral or bilateral adrenalectomy, Increase dose of Spironolactone Adrenalectomy is the treatment of choice for patients with primary adrenal hypertrophy. Spironolactone might be prescribed if high BP and hypokalemia. It is a drug that treats fluid retention and maintains potassium levels in the body. Corticosteroids should be reduced or tapered rather than increased. Diet high in carbohydrate and high in protein should be encouraged

A perioperative nurse is identified as being the scrub nurse for a surgical procedure. Which of the following is this nurses responsibilities during the surgery? (Select all that apply.) 1. Don surgical attire and personal protective equipment 2. Maintain the sterile field 3. Pass instruments and supplies to the surgeon 4. Prepare medication 5. Remove used instruments 6. Organize the sterile field for use

2, 3, 4 Maintain the sterile field, Pass instruments and supplies to the surgeon, prepare medication Responsibilities of the scrub nurse during a surgical procedure include maintaining the sterile field, passing instruments and supplies to the surgeon, and preparing medication. Donning surgical attire and organizing the sterile field are responsibilities done before the surgery begins. Removing used instruments are done after the surgery has concluded.

The postanesthesia care unit nurse is caring for clients with different types of wound drains. Which are the most common types of drains? (Select all that apply.) 1. Plantar drain 2. Penrose drain 3. Davol 4. Hemovac 5. Ostomy appliance 6. Chest tube collection device

2, 3, 4 Penrose drain, Davol, Hemovac The most common types of wound drains include the Penrose, Davol, and Hemovac. An ostomy appliance is not a postoperative wound drain. A chest tube collection device is not a postoperative wound drain.

A client is being evaluated for the diagnosis of gastroparesis. Which of the following will the nurse most likely assess in this client? (Select all that apply.) 1. Constipation 2. Gastroesophageal reflux 3. Feelings of fullness 4. Vomiting 5. Nausea 6. Anorexia

2, 3, 4, 5, 6 Gastroesophageal reflux, feelings of fullness, vomiting, nausea, anorexia Gastroparesis presents as anorexia, nausea and vomiting, feelings of fullness, and gastroesophageal reflux. Constipation is not a presenting sign of gastroparesis

A client with a diagnosis of diabetic ketoacidosis (DKA) is being treated in the emergency department. Which findings support this diagnosis? Select all that apply. 1. Increase in pH 2. Comatose state 3. Deep, rapid breathing 4. Decreased urine output 5. Elevated blood glucose level

2, 3, 5 -Comatose state -Deep, rapid breathing -Elevated blood glucose level Because of the profound deficiency of insulin associated with DKA, glucose cannot be used for energy and the body breaks down fat as a secondary source of energy. Ketones, which are acid by-products of fat metabolism, build up, and the client experiences a metabolic ketoacidosis. High serum glucose contributes to an osmotic diuresis and the client becomes severely dehydrated. If untreated, the client will become comatose due to severe dehydration, acidosis, and electrolyte imbalance. Kussmaul's respirations, the deep rapid breathing associated with DKA, is a compensatory mechanism by the body. The body attempts to correct the acidotic state by blowing off carbon dioxide (CO2), which is an acid. In the absence of insulin, the client will experience severe hyperglycemia. Option 1 is incorrect, because in acidosis the pH would be low. Option 4 is incorrect because a high serum glucose will result in an osmotic diuresis and the client will experience polyuria Focus on the subject, findings associated with DKA. Recall that the pathophysiology of DKA is the breakdown of fats for energy. The breakdown of fats leads to a state of acidosis. The high serum glucose contributes to an osmotic diuresis. Knowing the pathophysiology of DKA will aid in identification of the correct answers

The nurse teaches a client with diabetes mellitus about differentiating between hypoglycemia and ketoacidosis. The client demonstrates an understanding of the teaching by stating that a form of glucose should be taken if which symptoms develop? Select all that apply. 1. Polyuria 2. Shakiness 3. Palpitations 4. Blurred vision 5. Lightheadedness 6. Fruity breath odor

2, 3, 5 Shakiness, Palpitations, Lightheadedness Shakiness, palpitations, and lightheadedness are signs/symptoms of hypoglycemia and would indicate the need for food or glucose. Polyuria, blurred vision, and a fruity breath odor are manifestations of hyperglycemia Focus on the subject, the treatment of hypoglycemia. Think about its pathophysiology and the manifestations that occur. Recalling the signs and symptoms of hypoglycemia will direct you to the correct option

The nurse is assessing the endocrine system of an elderly client. Which of the following are considered age-related changes of this system? (Select all that apply.) 1. Increased estrogen in women 2. Increased production of antidiuretic hormone 3. Decreased testosterone in men 4. Increased pancreatic secretion of insulin 5. Smaller thyroid gland 6. Risk for osteoporosis

2, 3, 5, 6 Increased production of antidiuretic hormone, Decreased testosterone in men, Smaller thyroid gland, Risk for osteoporosis Age-related changes include a decreasing basal metabolic rate as a result of a smaller thyroid gland. There is an increased production of antidiuretic hormone, resulting in more dilute urine and polyuria. Other changes are that the pancreas secretes less insulin, estrogen decreases in women, and testosterone decreases in men. Because estrogen function decreases in females, there is an increased risk for osteoporosis.

The nurse is monitoring a client diagnosed with acromegaly who was treated with transsphenoidal hypophysectomy and is recovering in the intensive care unit. Which findings should alert the nurse to the presence of a possible postoperative complication? Select all that apply. 1. Anxiety 2. Leukocytosis 3. Chvostek's sign 4. Urinary output of 800 mL/hr 5. Clear drainage on nasal dripper pad

2, 4, 5 Leukocytosis, Urinary output of 800 mL/hr, Clear drainage on nasal dripper pad Acromegaly results from excess secretion of growth hormone, usually caused by a benign tumor on the anterior pituitary gland. Treatment is surgical removal of the tumor, usually with a sublingual transsphenoidal complete or partial hypophysectomy. The sublingual transsphenoidal approach is often through an incision in the inner upper lip at the gum line. Transsphenoidal surgery is a type of brain surgery, and infection is a primary concern. Leukocytosis, or an elevated white count, may indicate infection. Diabetes insipidus is a possible complication of transsphenoidal hypophysectomy. In diabetes insipidus there is decreased secretion of antidiuretic hormone, and clients excrete large amounts of dilute urine. Following transsphenoidal surgery, the nasal passages are packed and a dripper pad is secured under the nares. Clear drainage on the dripper pad is suggestive of a cerebrospinal fluid leak. The surgeon should be notified and the drainage should be tested for glucose. A cerebrospinal fluid leak increases the postoperative risk of meningitis. Anxiety is a nonspecific finding that is common to many disorders. Chvostek's sign is a test of nerve hyperexcitability associated with hypocalcemia and is seen as grimacing in response to tapping on the facial nerve. Chvostek's sign has no association with complications of sublingual transsphenoidal hypophysectomy. Focus on the subject, postoperative complications of sublingual transsphenoidal hypophysectomy. Knowing that infection, diabetes insipidus, and cerebrospinal fluid leak are possible complications will assist in determining the correct answer

A patient who has just had a total laryngectomy for cancer is being discharged. Which statement indicates the patient needs further teaching on care of the tracheostomy? 1. "I must avoid getting any objects in the stoma." 2. "I can take a shower when I get home." 3. "I can learn to speak with an electric larynx." 4. "I need to clean around the stoma every day."

2. "I can take a shower when I get home" The patient must be taught to avoid getting water into the stoma, preventing water direct access to the bronchial tree. Methods of covering the stoma to protect it from water must be taught. Bathing may be preferable to showering

A client with type 1 diabetes mellitus who takes NPH daily in the morning calls the nurse to report recurrent episodes of hypoglycemia with exercising. Which statement by the client indicates an adequate understanding of the peak action of NPH insulin and exercise? 1. "I should not exercise since I am taking insulin." 2. "The best time for me to exercise is after breakfast." 3. "The best time for me to exercise is mid- to late afternoon." 4. "NPH is a basal insulin, so I should exercise in the evening."

2. "The best time for me to exercise is after breakfast" Exercise is an important part of diabetes management. It promotes weight loss, decreases insulin resistance, and helps control blood glucose levels. A hypoglycemic reaction may occur in response to increased exercise, so clients should exercise either an hour after mealtime or after consuming a 10- to 15-g carbohydrate snack, and they should check their blood glucose level before exercising. Option 1 is incorrect because clients with diabetes should exercise, though they should check with their primary health care provider before starting a new exercise program. Option 3 in incorrect; clients should avoid exercise during the peak time of insulin. NPH insulin peaks at 4 to 12 hours; therefore, afternoon exercise takes place during the peak of the medication. Option 4 is incorrect; NPH insulin is an intermediate-acting insulin, not a basal insulin Focus on the subject, peak action of NPH insulin. Recalling that NPH insulin peaks at 4 to 12 hours and that exercise is beneficial for clients with diabetes will direct you to the correct option.

A morning dose of NPH insulin is given at 7:30 AM. What is the timeframe in which the nurse can expect it to peak? 1. 11:30 AM and 1:30 PM 2. 1:30 PM and 3:30 PM 3. 3:30 PM and 9:30 PM 4. 5:30 PM and 11:30 PM

2. 1:30 PM and 3:30 PM NPH insulin peaks in 6 to 8 hours.

A patient has a pancreatic tumor and history of alcoholism. The patient receives morphine 2 mg subcutaneously every 3 to 4 hours for treatment of pain associated with the tumor. After 2 days of receiving this dose every 4 hours, the patient tells the nurse that the medication is needed more frequently to control the pain. What is the nurse's best response to this situation? 1. The patient is becoming addicted to the morphine, and the medication should be administered less frequently than every 4 hours. 2. A tolerance to the morphine is developing, and the patient should receive the drug every 3 hours. 3. Administering the morphine every 3 hours will increase the patient's physical dependence on the drug. 4. Physical dependence should be avoided at all costs, and the drug should continue to be administered every 4 hours.

2. A tolerance to the morphine is developing, and the patient should receive the drug every 3 hours Tolerance occurs when a person who has been taking opioids becomes less sensitive to their analgesic properties (and usually side effects). Tolerance is characterized by the need for increasing doses to maintain the same level of pain relief.

When does the postoperative phase begin? 1. Admission of the patient to the OR 2. Admission of the patient to the PACU and ending when the patient is discharged to the unit or home 3. Admission of the patient to the PACU and ending with follow-up evaluation in the clinical setting or home 4. Admission to the PACU

2. Admission of the patient to the PACU and ending when the patient is discharged to the unit or home The postoperative phase does not just end when the patient is discharged to the unit or to home. The goal of postoperative management is to restore the patient to previous state or acceptable return of equilibrium. This phase includes teaching, promoting recovery, follow-up care, and referrals essential for recovery and rehabilitation.

Before an ABG is drawn from the patient's radial artery, what test should be performed? 1. Doppler flow test 2. Allen test 3. Babinski test 4. Ankle Brachial Index (ABI)

2. Allen test The Allen test is used to assess for adequate perfusion of the hand via the ulnar artery prior to puncturing the radial artery for an arterial blood gas. This test assesses patency of the ulnar artery and/or collateral circulation.

A client diagnosed with chronic obstructive pulmonary disease is experiencing pneumonia. The nurse applies oxygen at 2 L/min via nasal cannula. When the nurse leaves the room, a family member increases the oxygen to 5 L. Which complication may occur? 1. Angina 2. Apnea 3. Metabolic acidosis 4. Respiratory alkalosis

2. Apnea The COPD clients drive to breathe is hypoxia. Increasing the oxygen removes this drive and leads to apnea. Angina occurs because of decreased oxygen to the myocardial tissues. Neither respiratory alkalosis nor metabolic acidosis would occur with the increased oxygen level.

A patient who had abdominal surgery 5 days ago complains of sharp, throbbing abdominal pain that ranks 8 on a scale of 0 (no pain) to 10 (worst pain). The patient's pain rating has averaged at a 3/10 for the first 5 days postoperatively on epidural administration of hydromorphone. However, narcotic administration via epidural catheter was discontinued earlier this morning. Which of the following would be the nurse's first action? 1. an order to restart the epidural narcotic administration. 2. Assess the patient to rule out possible complications secondary to surgery. 3. Check the patient's chart to determine what additional pain medications can be administered. 4. Explain to the patient that his pain should not be this severe 5 days postoperatively.

2. Assess the patient to rule out possible complications secondary to surgery Usually of recent onset and commonly associated with a specific injury, acute pain indicates that damage or injury has occurred. Any change in the patient's condition requires an assessment by the nurse.

A client diagnosed with a lung abscess is being prescribed antibiotic therapy. Which of the following medications would be indicated if this client has a history of penicillin allergy? 1. Metronidazole 2. Clindamycin 3. Ampicillin 4. Steroid

2. Clindamycin Clients allergic to penicillin are often given clindamycin since this medication is not part of the penicillin family. Metronidazole and ampicillin should not be administered to this client. Steroid is not an antibiotic

Which should be included when teaching a newly diagnosed patient about the dietary management of diabetes? 1. Food intake should be decreased before exercise. 2. Consistency between food intake and activity is important. 3. Carbohydrates are strictly limited. 4. Insulin and other antidiabetic agents decrease the need for dietary management.

2. Consistency between food intake and activity is important Activity lowers the blood glucose; more food is needed with increased activity.

A client with diabetes mellitus demonstrates acute anxiety when admitted to the hospital for the treatment of hyperglycemia. What is the appropriate intervention to decrease the client's anxiety? 1. Administer a sedative. 2. Convey empathy, trust, and respect toward the client. 3. Ignore the signs and symptoms of anxiety, anticipating that they will soon disappear. 4. Make sure that the client is familiar with the correct medical terms to promote understanding of what is happening.

2. Convey empathy, trust, and respect toward the client Anxiety is a subjective feeling of apprehension, uneasiness, or dread. The appropriate intervention is to address the client's feelings related to the anxiety. Administering a sedative is not the most appropriate intervention and does not address the source of the client's anxiety. The nurse should not ignore the client's anxious feelings. Anxiety needs to be managed before meaningful client education can occur. Use therapeutic communication techniques to answer the question. Remember that the client's feelings are the priority. Keeping this in mind will direct you easily to the correct option

A client, diagnosed with arthritis, should be instructed to avoid the use of NSAIDs because of which of the following prescribed medications? 1. Penicillin 2. Coumadin 3. Digoxin 4. Diazide

2. Coumadin Persons at greatest risk for adverse reactions to NSAIDs include those who are prescribed warfarin (Coumadin) since the NSAID can increase the effects of the Coumadin and promote bleeding.

A clients chest tube has been accidentally dislodged while the client was being transferred from the bed to a stretcher. Which of the following should the nurse do to help this client? 1. Cover the site with occlusive petroleum jelly gauze and tape to four sides 2. Cover the site with occlusive petroleum jelly gauze and tape to three sides 3. Cover the site with occlusive petroleum jelly gauze and tape to two sides 4. Cover the site with occlusive petroleum jelly gauze and tape to one side

2. Cover the site with occlusive petroleum jelly gauze and tape to three sides In the case of accidental dislodging of the chest tube, the site should be covered with occlusive petroleum jelly gauze and taped on three sides to prevent the development of a tension pneumothorax. If the gauze is taped on all four sides, the client can develop a tension pneumothorax. Taping the gauze on one or two sides will not be effective to support this client and should not be done.

The nurse is assessing an adult patient experiencing hypoxia. Which of the following findings would be considered a late sign of hypoxia? 1. Confusion 2. Cyanosis 3. Drowsiness 4. Headache

2. Cyanosis Cyanosis is a late sign of hypoxia. Confusion, drowsiness, and headache are early signs

The emergency department nurse is assessing a client who has sustained a blunt injury to the chest wall. Which finding indicates the presence of a pneumothorax in this client? 1. A low respiratory rate 2. Diminished breath sounds 3. The presence of a barrel chest 4. A sucking sound at the site of injury

2. Diminished breath sounds This client has sustained a blunt or closed-chest injury. Basic symptoms of a closed pneumothorax are shortness of breath and chest pain. A larger pneumothorax may cause tachypnea, cyanosis, diminished breath sounds, and subcutaneous emphysema. Hyper-resonance also may occur on the affected side. A sucking sound at the site of injury would be noted with an open chest injury. Focus on the subject, a blunt chest injury. Noting the word blunt will assist in eliminating option 4, which describes a sucking chest wound injury. Knowing that in a respiratory injury increased respirations will occur will assist you in eliminating option 1. Option 3 can be eliminated because a barrel chest is a characteristic finding in a client with chronic obstructive pulmonary disease.

The nurse is caring for an 80-year-old patient following right hip surgery. When administering pain medication to this patient, what is important for the nurse to remember? 1. Elderly patients have a faster metabolism than their younger counterparts. 2. Elderly patients have an increased risk for drug toxicity. 3. Elderly patients have a lower incidence of chronic illness. 4. Elderly patients infrequently use OTC medications.

2. Elderly patients have an increased risk for drug toxicity Elderly patients have a sensitivity to medications and an increased risk for drug toxicity. They have slowed metabolism of medications, a higher incidence of chronic illness, and an increased use of prescription and OTC medications

The nurse is instructing a client diagnosed with type 2 diabetes mellitus on diagnostic tests used to evaluate the control of the disorder. The nurse should instruct the client on which of the following diagnostic tests that will provide this information? 1. Fasting plasma glucose 2. Glycosylated hemoglobin 3. Random plasma glucose 4. Two-hour oral glucose tolerance test

2. Glycosylated hemoglobin The glycosylated hemoglobin (hemoglobin A1c) test measures the amount of glucose attached to hemoglobin molecules and red blood cells over their life span of approximately 120 days. This test provides information about long-term control. The other options give current glucose information

The nurse assesses the patient with acute pharyngitis for what signs and symptoms that indicate a complication of GAS pharyngitis? 1. Pain and spasm of the lower leg muscles 2. Gross hematuria, edema, and hypertension 3. Reduced visual fields, blurred vision 4. Insomnia, heartburn, and abdominal distension

2. Gross hematuria, edema, and hypertension Acute poststreptococcal glomerulonephritis (APSGN) is a complication that follows roughly 10 days after the onset of streptococcal infection and results in temporary kidney failure. APSGN is characterized by the rapid onset of gross hematuria, edema (leading to respiratory distress and pulmonary edema), and hypertension

A client complains of nervousness and palpitations. Upon assessing the patient's heart rate, the nurse notes a heart rate of 120 bpm. Which of the following endocrine disorders is associated with palpitations and increased heart rate? 1. Hypothyroidism 2. Hyperthyroidism 3. SIADH 4. Hypoparathyroidism

2. Hyperthyroidism Hyperthyroidism increases the heart rate and palpitations. Thyroid hormone increases the cardiac contractility, cardiac output, and heart rate.

The nurse has instructed a client about type 2 diabetes mellitus. Which of the following statements would indicate the client understands the instructions? 1. It happens to everyone who has gained weight 2. I have to watch what I eat and exercise 3. I will never have to take insulin 4. The cells that make insulin were destroyed

2. I have to watch what I eat and exercise Persons with type 2 diabetes control their blood glucose levels with diet, exercise, and medications. Type 1 diabetes mellitus is characterized by a destruction of beta cells. Not every person who gains weight develops diabetes mellitus. Insulin is not necessary for the client diagnosed with type 2 diabetes at first, but as the beta cells continue to deteriorate, insulin may be necessary.

The nurse is instructing a client diagnosed with type 2 diabetes mellitus on daily foot care. Which of the following statements indicate the client needs further instruction? 1. I will check my feet every day 2. I will cut my toenails with scissors 3. I will keep my appointments with my podiatrist 4. I will make sure my shoes fit

2. I will cut my toenails with scissors Clients and their family members knowledge and practice of foot care should be assessed regularly. Clients should be instructed to wash their feet daily with warm water and mild soap. The feet should be patted dry, particularly between the toes. The feet should be examined daily for cuts, blisters, and reddened areas. Toenails should be cut with clippers, not scissors. The shoes of a client diagnosed with type 2 diabetes mellitus should fit properly to prevent foot problems.

The nurse is preparing a plan of care for a client with diabetes mellitus who has hyperglycemia. The nurse places priority on which client problem? 1. Lack of knowledge 2. Inadequate fluid volume 3. Compromised family coping 4. Inadequate consumption of nutrients

2. Inadequate fluid volume An increased blood glucose level will cause the kidneys to excrete the glucose in the urine. This glucose is accompanied by fluids and electrolytes, causing an osmotic diuresis leading to dehydration. This fluid loss must be replaced when it becomes severe. Options 1, 3, and 4 are not related specifically to the information in the question Note the strategic word, priority, and focus on the information in the question. Use Maslow's Hierarchy of Needs theory. The correct option indicates a physiological need and is the priority. Options 1, 3, and 4 are problems that may need to be addressed after providing for the priority physiological needs.

The nurse is caring for a client diagnosed with ARDS. In evaluating the use of PEEP, what outcome would the nurse expect to find? 1. Increased ventilation-perfusion mismatch 2. Increased FRC 3. Decreased intrathoracic pressure 4. Decreased FRC

2. Increased FRC The use of PEEP helps increase functional residual capacity (FRC) and reverse alveolar collapse by keeping the alveoli open, resulting in improved arterial oxygenation and a reduction in the severity of the ventilation--perfusion imbalance. The use of positive-pressure ventilation increases intrathoracic pressure and causes a decrease in preload to the heart.

A client diagnosed with type 2 diabetes mellitus becomes diaphoretic and irritable during exercise. The blood glucose level at this time is 53 mg/dL. Which of the following should the client be instructed to do when this occurs? 1. Ingest 5 to 10 g of a simple carbohydrate 2. Ingest 10 to 15 g of a simple carbohydrate 3. Ingest 15 to 25 g of a simple carbohydrate 4. Call paramedics

2. Ingest 10 to 15 g of a simple carbohydrate If the client becomes hypoglycemic during exercise, the client should be instructed to stop and monitor the blood glucose level every 15 minutes until the level is greater than 89 mg/dL. The client should ingest 15 grams of a carbohydrate such as milk, juice, soft drink, or glucose tablets. The treatment can be repeated in 15 minutes if ineffective.

A 70-year-old female patient is admitted to the unit with enlargement of thyroid gland, hypertension, high TSH levels, and bulging eyes. What nursing intervention is most appropriate for this patient? 1. Providing a blanket 2. Instilling eye ointment 3. Providing a warm bath 4. Keep room temperature comfortable at 85°F

2. Instilling eye ointment A patient with Graves' disease is likely to show signs of exophthalmos (protrusion of eyeball that exposes the cornea). Instillation of eye ointment is necessary to minimize corneal damage. Patients with Graves' disease perspire easily and we should keep room temperature cool rather than warm

A nurse is caring for a postoperative patient who had spinal anesthesia. The patient complains of headache. What action should the nurse take? 1. Lower the head of the patient's bed. 2. Keep the patient lying flat, maintain a quiet environment, and keep the patient hydrated. 3. Encourage the patient to lay on his right side. 4. Do nothing, as this is a normal response to the spinal anesthesia.

2. Keep the patient lying flat, maintain a quiet environment, and keep the patient hydrated Headache may be an after-effect of spinal anesthesia. Factors related to headache include, size of the spinal needle used, the leakage of the fluid from subarachnoid space through the puncture site and the patients hydration status. Measures that increase cerebrospinal pressure are helpful in relieving headache

The nurse is instructing a client diagnosed with type 2 diabetes mellitus on dietary intake. Which of the following statements indicates that the client understands the instructions? 1. It's okay to skip a meal if I make it up later 2. Keeping to the diet plan will keep my blood sugars at a regular level 3. When I am in a hurry, I should take my medications without testing 4. When I go out to dinner, it's okay to share a couple of bottles of wine

2. Keeping to the diet plan will keep my blood sugars at a regular level The diet plan is individualized for each client. The food plan will have an emphasis on maintaining blood glucose levels, lowering blood pressure, and reducing weight since there is a high incidence of obesity in clients with type 2 diabetes. Alcohol can be part of a diet plan if in moderation. Sharing a couple of bottles of wine would not be alcohol in moderation. The food plan is combined with exercise, blood glucose testing, and medications (if needed). The client should be instructed to not skip meals. The client should be instructed to not take any medication prior to testing. The client should be instructed that alcohol intake should be in moderation.

The nurse is instructing a client on the speed in which some insulins take effect. During these instructions, the nurse should include that which of the following insulins has the fastest peak? 1. Glargine 2. Lispro 3. NPH 4. Regular

2. Lispro Lispro (Humalog) is classified as an ultra-short-acting insulin that peaks in 30 to 90 minutes after subcutaneous injection. Regular is a short-acting insulin that peaks in 2 to 4 hours. NPH peaks in 4 to 12 hours. Glargine takes effect in 2 to 4 hours and has no peak.

A client is admitted to an emergency department, and a diagnosis of myxedema coma is made. Which action should the nurse prepare to carry out initially? 1. Warm the client. 2. Maintain a patent airway. 3. Administer thyroid hormone. 4. Administer fluid replacement.

2. Maintain a patent airway Myxedema coma is a rare but serious disorder that results from persistently low thyroid production. Coma can be precipitated by acute illness, rapid withdrawal of thyroid medication, anesthesia and surgery, hypothermia, and the use of sedatives and opioid analgesics. In myxedema coma, the initial nursing action is to maintain a patent airway. Oxygen should be administered, followed by fluid replacement, keeping the client warm, monitoring vital signs, and administering thyroid hormones by the intravenous route Note the strategic word, initially. All the options are appropriate interventions, but use the ABCs—airway, breathing, and circulation—in selecting the correct option

The nurse wants to reduce the stress level for a preoperative client. Which of the following communication techniques can the nurse use to achieve this result? 1. Allow the client to be alone before the surgery 2. Observe and ask the client if there is anything that can be done to help reduce her anxiety 3. Refer to the client by her first name 4. Make tasteful jokes or comments to help the client laugh

2. Observe and ask the client if there is anything that can be done to help reduce her anxiety Strategies to reduce preoperative stress include observing and asking the client if there is anything that can be done to help reduce her anxiety. Leaving the client alone before the surgery will not help reduce stress. Referring to the client by her first name might be considered unprofessional and should not be done. Making jokes is also not a professional behavior and should not be done by the nurse

The nurse is preparing to give a bed bath to an immobilized client with tuberculosis. The nurse should wear which items when performing this care? 1. Surgical mask and gloves 2. Particulate respirator, gown, and gloves 3. Particulate respirator and protective eyewear 4. Surgical mask, gown, and protective eyewear

2. Particulate respirator, gown, and gloves The nurse who is in contact with a client with tuberculosis should wear an individually fitted particulate respirator. The nurse also would wear gloves as per standard precautions. The nurse wears a gown when the possibility exists that the clothing could become contaminated, such as when giving a bed bath. Focus on the subject, precautions when caring for the client with tuberculosis. Think about the nurse's task, a bed bath. Knowing that the nurse should wear a particulate respirator eliminates options 1 and 4. Knowledge of basic standard precautions directs you to the correct option.

The nurse assesses a client's surgical incision for signs of infection. Which finding by the nurse would be interpreted as a normal finding at the surgical site? 1. Red, hard skin 2. Serous drainage 3. Purulent drainage 4. Warm, tender skin

2. Serous drainage Serous drainage is an expected finding at a surgical site. The other options indicate signs of wound infection. Signs and symptoms of infection include warm, red, and tender skin around the incision. Wound infection usually appears 3 to 6 days after surgery. The client also may have a fever and chills. Purulent material may exit from drains or from separated wound edges. Infection may be caused by poor aseptic technique or a contaminated wound before surgical exploration; existing client conditions such as diabetes mellitus or immunocompromise may place the client at risk. Focus on the subject, normal findings in the postoperative period. Eliminate options 1, 3, and 4 because they are comparable or alike and are manifestations of infection

An elderly client is scheduled for a surgical procedure. The nurse realizes that the outcome of the clients operation will depend upon the clients: 1. Age 2. Severity of illnesses 3. Nutritional status 4. Activity status

2. Severity of illnesses Severity of illness is a much better predictor of outcome of surgery when compared to age. Nutritional status and activity status would be characteristics that are associated with severity of illness

A client is diagnosed with fractured ribs. Which of the following should the nurse instruct this client? 1. Engage in routine activities of daily living after taking pain medication 2. Splint the rib cage when deep breathing and coughing 3. Restrict fluids 4. Stay on bed rest until the ribs heal

2. Splint the rib cage when deep breathing and coughing Nursing care for a client recovering from fractured ribs include splinting the rib cage when deep breathing and coughing. The client should be encouraged to avoid dangerous activities when taking pain medication. Fluids should not be restricted. Bed rest would not be necessary for fractured ribs

A client is scheduled for a thyroid scan. Which of the following should the nurse instruct the client regarding this diagnostic test? 1. Eliminate all salt in the diet 2. Take nothing by mouth after midnight if I-131 is being used during the test 3. Continue taking thyroid medication as prescribed 4. Take nothing by mouth for 45 minutes after receiving intravenous technetium for the test

2. Take nothing by mouth after midnight if I-131 is being used during the test Depending upon the medium being used, instructions to a client prior to having a thyroid scan may differ; however, if the client is having an oral dose of I-131 for the test, the client should be instructed to take nothing by mouth after midnight. Clients who are prescribed medications with iodine may be instructed to stop the medications for 2 weeks prior to the scan. If receiving intravenous technetium for the scan, the client does not need to be kept on a nothing by mouth order. The client should not be instructed to eliminate all salt from the diet. The client may be instructed to discontinue all thyroid medication for 4 to 6 weeks prior to the scan

The nurse is caring for a client after hypophysectomy and notes clear nasal drainage from the client's nostril. The nurse should take which initial action? 1. Lower the head of the bed. 2. Test the drainage for glucose. 3. Obtain a culture of the drainage. 4. Continue to observe the drainage

2. Test the drainage for glucose After hypophysectomy, the client should be monitored for rhinorrhea, which could indicate a cerebrospinal fluid leak. If this occurs, the drainage should be collected and tested for the presence of cerebrospinal fluid. Cerebrospinal fluid contains glucose, and if positive, this would indicate that the drainage is cerebrospinal fluid. The head of the bed should remain elevated to prevent increased intracranial pressure. Clear nasal drainage would not indicate the need for a culture. Continuing to observe the drainage without taking action could result in a serious complication. Note the strategic word, initial, and determine whether an abnormality exists. This indicates that an action is required. Option 1 can be eliminated first by recalling that this action can increase intracranial pressure. Option 3 can be eliminated also, because the drainage is clear. Because an action is required, eliminate option 4.

A client recovering from surgery tells the nurse that she is nauseated and is experiencing an increase in pain. Which of the following does this clients symptoms suggest to the nurse? 1. The client is becoming dependent upon the pain medication 2. The clients pain threshold is lower when experiencing nausea 3. The client is experiencing withdrawal symptoms from pain medication 4. The client is experiencing referred pain

2. The clients pain threshold is lower when experiencing nausea Pain threshold is influenced by nausea, fatigue, and lack of sleep. The client experiencing an increase in pain during nausea is demonstrating an alteration in the pain threshold. The client is not becoming dependent upon the pain medication. The client is not experiencing withdrawal symptoms. The client is also not experiencing referred pain.

A 70-year-old elderly patient is admitted to the preoperative unit for a liver resection that is scheduled for 6 hours of surgery. What is one of the basic principles that should guide the preoperative nurse's assessment? 1. Elderly patients do not experience as much preoperative anxiety as younger patients. 2. The elderly patient has less physiologic reserve than the younger patient. 3. Elderly patients experience less pain. 4. Preoperative pain assessment and teaching should occur following the procedure as the elderly patient may not retain the information.

2. The elderly patient has less physiologic reserve than the younger patient Preoperative pain assessment and teaching are important to the elderly patient. These patients may have a combination of chronic illnesses and health issues in addition to the specific one for which surgery is indicated. Health care staff must remember that the hazards of surgery for the aged are proportional to the number and severity of coexisting health problems and the nature and duration of the operative procedure. It is, therefore, important to be guided by the principle that the elderly has less physiologic reserve (the ability of an organ to return to normal after a disturbance in its equilibrium).

Surgical wound healing occurs in: 1. Two phases: inflammatory and maturation 2. Three phases: inflammatory, proliferative, and maturation 3. First-, second-, and third-intention wound healing 4. First and proliferative phase

2. Three phases: inflammatory, proliferative, and maturation Wounds heal in three phases and the duration of the phases can vary in length. First- second- and third-intention wound healing is the mechanical process of wound healing.

A patient will be going home with a tracheal stoma. What information does the nurse include in the discharge teaching for this patient? 1. Water sports are permitted. 2. Wear a medical identification bracelet. 3. Mouth-to-mouth ventilation should be performed in an emergency. 4. Fifteen minutes of strenuous exercise will improve overall strength.

2. Wear a medical identification bracelet . The patient needs to wear or carry medical identification to alert medical personnel to the special requirements of resuscitation should this need arise. If resuscitation is needed, direct mouth-to-stoma ventilation should be provided. Water sports are prohibited because a person with a tracheal stoma can drown without submerging his or her face. It is important that strenuous exercise and fatigue be avoided because the patient will have more difficulty speaking when tired, which can be discouraging

A client is observed holding a pillow over the abdominal region with both knees flexed in a side-lying position. Vital signs assessment reveals an elevated blood pressure and heart rate. Which of the following should the nurse say to this client? 1. Can I get you anything? 2. Would you like something for pain? 3. You look comfortable. 4. Your blood pressure is up

2. Would you like something for pain? Sympathetic responses to pain include elevated blood pressure and heart rate. And since the client is hugging a pillow over the abdominal region with both knees flexed in a side-lying position, the best thing for the nurse to say to this client is Would you like something for pain? The other responses are incorrect because they do not acknowledge that the client is experiencing pain

A client is experiencing a disorder to the anterior pituitary gland. The nurse realizes that all of the following hormones will be affected by this disorder EXCEPT: 1. adrenocorticotropic hormone 2. antidiuretic hormone 3. melanocyte-stimulating hormone 4. luteinizing hormone

2. antidiuretic hormone Antidiuretic hormone is stored by the posterior pituitary. The other choices are under the regulation of the anterior pituitary gland and would be affected by a disorder in this area

The preoperative nurse cares for the client until the client progresses into the intraoperative phase of care which begins when the client: 1. signs the surgical consent form 2. arrives at the surgical suite doors 3. is transferred to the postanesthetic care unit 4. accepts that surgery is pending

2. arrives at the surgical suite doors The preoperative period ends and the intraoperative period begins when the patient and family are at the door to the surgical suites. Intraoperative care does not begin when the client signs the surgical consent form, is transferred to the postanesthesia care unit, or accepts that surgery is pending

The nurse, assessing a clients breath sounds, has the stethoscope placed over the second intercostal space next to the sternum. The sound the nurse is most likely going to hear would be: 1. vesicular 2. bronchovesicular 3. bronchial 4. absent

2. bronchovesicular Bronchovesicular breath sounds are loud and harsh and are most likely heard over the trachea. Vesicular breath sounds can be heard anywhere over the lung fields. Bronchial sounds are only normally heard over the trachea. They are loud and harsh in quality, high-pitched, and sound hollow. Absent breath sounds can also be heard throughout the lung fields.

The nurse is to collect a stool specimen from a client diagnosed with cystic fibrosis. The nurse would expect to see: 1. black, tarry stool 2. bulky, foul-smelling stool 3. clay-colored stool 4. green stool

2. bulky, foul-smelling stool Bulky, foul-smelling stool is characteristic of clients diagnosed with cystic fibrosis as a result of malabsorption. Black, tarry stool can be observed in a client with upper gastrointestinal bleeding. Clay-colored stool can indicate bile obstruction. Green stool may indicate gastrointestinal infection.

A client recovering from anesthesia in the care unit has an artificial airway. The nurse knows the purpose of an artificial airway is to: 1. keep the mouth open 2. keep the tongue from blocking the airway 3. keep the client from vomiting 4. allow the client to talk

2. keep the tongue from blocking the airway The artificial airway ensures that the tongue does not block the upper airway. An artificial airway may or may not keep the mouth open. An artificial airway will not prevent the client from vomiting and is not used to facilitate client communication

A client with a nasogastric tube connected to low continuous suction has the following arterial blood gas (ABG) results: pH 7.49, PaO2 91, PaCO2 42, and HCO3 31. Interpreting these result, the nurse concludes that the client is in: 1. metabolic acidosis 2. metabolic alkalosis 3. respiratory acidosis 4. respiratory alkalosis

2. metabolic alkalosis Because the pH is greater than 7.45, this is not an acidosis. The PaCO2 is within normal limits. The HCO3is elevated. An elevated pH and HCO3 indicates metabolic alkalosis.

The nurse, caring for a postoperative client, will apply supplemental oxygen because: 1. the client needs it 2. of anesthetic gasses in the lungs 3. it helps control blood pressure 4. it helps with wound healing

2. of anesthetic gasses in the lungs Postoperative clients require supplemental oxygen because they may still be retaining anesthetic gasses in the lungs. The client will not be able to state that they need oxygen. Oxygen will not control blood pressure nor will it help with wound healing

A client is diagnosed with stage I mild, chronic obstructive pulmonary disease. Which of the following assessment findings will support this diagnosis? (Select all that apply.) 1. Chronic cough 2. Sputum production 3. Forced expiratory volume in 1 second of greater than 80% 4. Mild airflow limitations 5. Extreme dyspnea on exertion 6. Right-sided heart failure

3, 4 Forced expiratory volume in 1 second of greater than 80%, Mild airflow limitations In stage I mild chronic obstructive pulmonary disease, the client will demonstrate mild airflow limitations and have a forced expiratory volume in 1 second of greater than 80%. Chronic cough and sputum production are signs of stage 0 of the disease. Extreme dyspnea on exertion and right-sided heart failure are indications of stage III severe chronic obstructive pulmonary disease.

A client is scheduled for diagnostic tests to evaluate the adrenal glands. Which of the following will most likely be included in these tests? (Select all that apply.) 1. Vasopressin level 2. Urine specific gravity 3. Cortisol level 4. Dexamethasone suppression test 5. Progesterone assay 6. Aldosterone assay

3, 4, 5, 6 Cortisol level, Dexamethasone suppression test, progesterone assay, aldosterone assay Diagnostic tests used to evaluate the status of the adrenal glands include cortisol level, dexamethasone suppression test, progesterone assay, and aldosterone assay. Vasopressin level and urine specific gravity are used to assess the pituitary gland.

The nurse is preparing a client with a new diagnosis of hypothyroidism for discharge. The nurse determines that the client understands discharge instructions if the client states that which signs and symptoms are associated with this diagnosis? Select all that apply. 1. Tremors 2. Weight loss 3. Feeling cold 4. Loss of body hair 5. Persistent lethargy 6. Puffiness of the face

3, 4, 5, 6 Feeling cold, hair loss, lethargy, and facial puffiness are signs of hypothyroidism. Tremors and weight loss are signs of hyperthyroidism Focus on the subject, signs and symptoms associated with hypothyroidism. Options 1 and 2 can be eliminated if you remember that in hypothyroidism there is an undersecretion of thyroid hormone that causes the metabolism to slow down

A client is demonstrating signs of peritonsillar abscess. Which of the following will the nurse most likely assess in this client? (Select all that apply.) 1. Bradypnea 2. Drop in blood pressure 3. Hot potato voice 4. Trismus 5. Dysphagia 6. Sore throat

3, 4, 5, 6 Hot potato voice, trismus, dysphagia, sore throat Assessment findings consistent with peritonsillar abscess include: hot potato voice; trismus, or difficulty fully opening the mouth; dysphagia, or painful swallowing; and sore throat. Bradypnea and drop in blood pressure are not assessment findings consistent with peritonsillar abscess

A preoperative client expresses anxiety to the nurse about upcoming surgery. Which response by the nurse is most likely to stimulate further discussion between the client and the nurse? 1. "If it's any help, everyone is nervous before surgery." 2. "I will be happy to explain the entire surgical procedure to you." 3. "Can you share with me what you've been told about your surgery?" 4. "Let me tell you about the care you'll receive after surgery and the amount of pain you can anticipate."

3. "Can you share with me what you've been told about your surgery?" Explanations should begin with the information that the client knows. By providing the client with individualized explanations of care and procedures, the nurse can assist the client in handling anxiety and fear for a smooth preoperative experience. Clients who are calm and emotionally prepared for surgery withstand anesthesia better and experience fewer postoperative complications. Option 1 does not focus on the client's anxiety. Explaining the entire surgical procedure may increase the client's anxiety. Option 4 avoids the client's anxiety and is focused on postoperative care. Note that the client expresses anxiety. Use therapeutic communication techniques. Note that the question contains strategic words, most likely, and also note the words stimulate further discussion. Also use the steps of the nursing process. The correct option addresses assessment and is the only therapeutic response

The nurse has conducted preoperative teaching for a client scheduled for surgery in 1 week. The client has a history of arthritis and has been taking acetylsalicylic acid. The nurse determines that the client needs additional teaching if the client makes which statement? 1. "Aspirin can cause bleeding after surgery." 2. "Aspirin can cause my ability to clot blood to be abnormal." 3. "I need to continue to take the aspirin until the day of surgery." 4. "I need to check with my doctor about the need to stop the aspirin before the scheduled surgery."

3. "I need to continue to take the aspirin until the day of surgery." Antiplatelets alter normal clotting factors and increase the risk of bleeding after surgery. Aspirin has properties that can alter platelet aggregation and should be discontinued at least 48 hours before surgery. However, the client should always check with his or her surgeon regarding when to stop taking the aspirin when a surgical procedure is scheduled. Options 1, 2, and 4 are accurate client statements. Note the strategic words, needs additional teaching. These words indicate a negative event query and that you need to select the incorrect client statement. Eliminate options 1 and 2 first because they are comparable or alike. From the remaining options, recalling that aspirin has properties that can alter platelet aggregation will direct you to the correct option

Which statement indicates further teaching is required for the patient diagnosed with obstructive sleep apnea (OSA)? 1. "The CPAP machine will help me be more awake during the day." 2. "If I exercise and lose weight, I may not need to use CPAP." 3. "I will use the CPAP machine only when I really need to sleep well." 4. "The CPAP helps to keep my airway open when I sleep."

3. "I will use the CPAP machine only when I really need to sleep well" The patient must be taught that OSA is a condition that is treated but not cured by the use of CPAP. Failure to use CPAP at night will allow the nasopharyngeal structures to collapse again, impairing the airway and potentially leading to serious complications.

The nurse, caring for a client recovering from surgery, is monitoring the urine output and will notify the surgeon if the output falls below: 1. 10 mL/hr 2. 20 mL/hr 3. 30 mL/hr 4. 50 mL/hr

3. 30 mL/hr With proper renal function, the kidneys will produce a minimum of 30 mL of urine per hour. A urine output of 10 or 20 mL/hr should be reported to the physician. A urine output of 50 mL/hr does not need to be reported.

The nurse should instruct a client that the length of time insulin can be stored at room temperature is: 1. 2 weeks 2. 3 weeks 3. 4 weeks 4. 5 weeks

3. 4 weeks An insulin vial that is currently in use can be stored at room temperature as long as 4 weeks. The other choices are incorrect lengths of time to store insulin.

A client undergoes a purified protein derivative (PPD) test. The test should be read: 1. Immediately after the test 2. 24 to 48 hours after the test 3. 48 to 72 hours after the test 4. anytime after 72 hours

3. 48 to 72 hours after the test A small amount of tuberculin is injected directly under the skin at the site and is read 48 to 72 hours after the test. The test should not be read immediately afterwards or within 24 to 48 hours. If the test is read after 72 hours, the test may need to be repeated.

The nurse is reviewing clients for risk factors in the development of pneumonia. Which of the following clients would be at the highest risk for developing this disorder? 1. A 48-year-old client experiencing menopause 2. An 18-year-old client with abdominal pain 3. A 23-year-old client diagnosed with sickle-cell anemia and a cough 4. A 3-year-old client with fever

3. A 23-year-old client diagnosed with sickle-cell anemia and a cough High-risk groups for acquiring pneumonia are people with diabetes, infants 6- to 23-months old, and those with a chronic illness such as sickle-cell anemia. Menopause and abdominal pain are not symptoms associated with pneumonia. Fever in a 3-year-old client could be caused by many disorders and not necessarily pneumonia.

The nurse is performing an assessment on a client with pheochromocytoma. Which assessment data would indicate a potential complication associated with this disorder? 1. A urinary output of 50 mL/hr 2. A coagulation time of 5 minutes 3. A heart rate that is 90 beats per minute and irregular 4. A blood urea nitrogen level of 20 mg/dL (7.1 mmol/L)

3. A heart rate that is 90 beats per minute and irregular Pheochromocytoma is a catecholamine-producing tumor usually found in the adrenal medulla, but extra-adrenal locations include the chest, bladder, abdomen, and brain; it is typically a benign tumor but can be malignant. Excessive amounts of epinephrine and norepinephrine are secreted. The complications associated with pheochromocytoma include hypertensive retinopathy and nephropathy, myocarditis, increased platelet aggregation, and stroke. Death can occur from shock, stroke, kidney failure, dysrhythmias, or dissecting aortic aneurysm. An irregular heart rate indicates the presence of a dysrhythmia. A coagulation time of 5 minutes is normal. A urinary output of 50 mL/hr is an adequate output. A blood urea nitrogen level of 20 mg/dL (7.1 mmol/L) is a normal finding. Use the ABCs—airway, breathing, and circulation. An irregular heart rate is associated with circulation. In addition, knowing the normal hourly expectations associated with urinary output and the normal laboratory values for coagulation time and blood urea nitrogen level assists in selection of the correct option.

Which does the nurse recognize as an early indicator of nephropathy? 1. Hematuria 2. Glycosuria 3. Albuminuria 4. Polyuria

3. Albuminuria Damaged kidneys start "leaking" protein in the form of microalbumin. Normally, protein is not found in urine.

The nurse is concerned that a client may have an undocumented allergy to latex when which of the following is assessed? 1. Recent episode of appendicitis 2. Recovered from bronchitis 3 months ago 3. Allergy to specific foods 4. Does not like to wear wool clothing

3. Allergy to specific foods Risk factors for latex allergy include a history of allergies, for example, food allergies or contact dermatitis (eczema). Appendicitis and bronchitis do not increase the clients risk of a latex allergy. The clients not wearing wool clothing does not increase the clients risk of a latex allergy

The nurse is providing preoperative teaching for the patient who is to undergo a total laryngectomy. Which nursing intervention is most important? 1. Having the patient restrict food and fluids 2. Teaching the care required for the tracheostomy tube 3. Assessing the patient's ability to communicate postoperatively 4. Demonstrating cough and deep-breathing exercises

3. Assessing the patient's ability to communicate postoperatively The patient is having the vocal cords removed and will be unable to speak. The highest priority is to establish a means of communication for the postoperative period

The nurse is assessing a client diagnosed with hyperaldosteronism. Which of the following would take the least priority during this period? 1. Assessment of breath sounds 2. Cardiac monitoring 3. Assistance with activities of daily living (ADLs) 4. Review of electrolyte levels

3. Assistance with activities of daily living (ADLs) The first priority for the nurse is to monitor cardiac and respiratory status. Cardiac status can be impaired because of changes in potassium levels, and fluid balance can be impaired because of sodium, affecting the respiratory status. After the client is stabilized, the nurse can assist the client with activities of daily living.

A client has just been admitted to the nursing unit following thyroidectomy. Which assessment is the priority for this client? 1. Hoarseness 2. Hypocalcemia 3. Audible stridor 4. Edema at the surgical site

3. Audible stridor Thyroidectomy is the removal of the thyroid gland, which is located in the anterior neck. It is very important to monitor airway status, as any swelling to the surgical site could cause respiratory distress. Although all of the options are important for the nurse to monitor, the priority nursing action is to monitor the airway Note the strategic word, priority. Use the ABCs—airway, breathing, and circulation, to assist in directing you to the correct option.

The nurse is caring for a client after a bronchoscopy and biopsy. Which finding, if noted in the client, should be reported immediately to the primary health care provider? 1. Dry cough 2. Hematuria 3. Bronchospasm 4. Blood-streaked sputum

3. Bronchospasm If a biopsy was performed during a bronchoscopy, blood-streaked sputum is expected for several hours. Frank blood indicates hemorrhage. A dry cough may be expected. The client should be assessed for signs of complications, which would include cyanosis, dyspnea, stridor, bronchospasm, hemoptysis, hypotension, tachycardia, and dysrhythmias. Hematuria is unrelated to this procedure. Note the strategic word, immediately. Eliminate option 2 first because it is unrelated to the procedure. Next, eliminate option 1, because a dry cough may be expected. Noting that a biopsy has been performed will assist in eliminating option 4, because blood-streaked sputum would be expected. Note that the correct option relates to the airway.

An elderly woman complaining of weight gain, depression, and lethargy is diagnosed with hypothyroidism, and thyroid replacement is prescribed. During initiation of thyroid replacement therapy for the patient, the priority assessment for the nurse is to evaluate which of the following? 1. Mental status 2. Nutritional status 3. Cardiovascular function 4. Bowel function

3. Cardiovascular function Hypothyroidism is associated with coronary-related diseases such as elevated serum cholesterol, atherosclerosis, and coronary artery disease (CAD). When thyroid hormone is administered, the myocardial oxygen demand increases without increasing the myocardial oxygen supply; therefore, patients should be monitored for cardiac complications such as chest pain and congestive heart failure. Patients may also be treated for angina or arrhythmias due to the release of catecholamine's that may be activated during thyroid replacement therapy.

A client has experienced pulmonary embolism. The nurse should assess for which symptom, which is most commonly reported? 1. Hot, flushed feeling 2. Sudden chills and fever 3. Chest pain that occurs suddenly 4. Dyspnea when deep breaths are taken

3. Chest pain that occurs suddenly The most common initial symptom in pulmonary embolism is chest pain that is sudden in onset. The next most commonly reported symptom is dyspnea, which is accompanied by an increased respiratory rate. Other typical symptoms of pulmonary embolism include apprehension and restlessness, tachycardia, cough, and cyanosis. Note the strategic word, most. Because pulmonary embolism does not result from an infectious process or an allergic reaction, eliminate options 1 and 2 first. To select between the correct option and option 4, look at them closely. Option 4 states dyspnea when deep breaths are taken. Although dyspnea commonly occurs with pulmonary embolism, dyspnea is not associated only with deep breathing. Therefore, eliminate option 4.

The nurse assesses an area of drainage on the dressing of a postanesthesia care clients surgical wound. Which of the following should the nurse do? 1. Call the surgeon right away 2. Cover the dressing with a new dressing 3. Circle the area and mark it with the date and time 4. Pass it off to the next shift

3. Circle the area and mark it with the date and time If any drainage is showing on the dressing, the nurse is to circle the area and mark it with the date and time. The surgeon does not need to be phoned unless excessive bleeding or hematoma formation has occurred. The dressing does not need to be covered with a new dressing. The nurse should not pass this finding off to the next shift

The nurse is instructing a family member on how to change a clients postoperative wound dressing at home. Which of the following should be included in these instructions? 1. Wear gloves to remove the old dressing 2. Wear sterile gloves to apply the new dressing 3. Clean hands prior to applying the new dressing 4. Reposition the new dressing after application

3. Clean hands prior to applying the new dressing If the client is to change the dressing at home, there is no need to wear gloves when the old dressing is removed. Clean hands are sufficient to apply the new dressing. Sterile gloves are not needed to apply the new dressing. Once the new dressing has been placed over the wound, it should be left alone and not repositioned.

.The nurse is concerned that a client scheduled for surgery will be at risk for hypothermia. Which of the following did the nurse assess in this client to determine the risk? 1. Client is a vegetarian 2. Client exercises 5 days a week for 30 minutes 3. Client has a history of congestive heart failure 4. Client is 48 years old

3. Client has a history of congestive heart failure Clients at risk for hypothermia include the very young, the very old, those with a history of heart disease, those with a bleeding tendency, having complex surgery, and having surgery on a large body area that will be exposed. Being a vegetarian or exercising does not predispose a client to developing hypothermia during surgery.

The nurse should suspect that a client has an endocrine disorder when which of the following findings is assessed regarding the integumentary status? 1. Freckles 2. Presence of moles 3. Course hair 4. Even skin tone

3. Course hair Evidence of an endocrine disorder that can be assessed through the integumentary status would include hair loss, dry skin, course hair, brittle nails, or changes in pigmentation. Freckles, the presence of moles, or even skin tone are not evidence of an endocrine disorder.

A patient presents to the emergency room with pulmonary edema. The nurse expects to hear what adventitious sound? 1. Decreased breath sounds 2. Inspiratory and expiratory wheezing 3. Crackles 4. Friction rub

3. Crackles Crackles are associated with heart failure

A client, complaining of weight gain, has thin extremities, a buffalo hump, and a protruding abdomen. The nurse realizes that this client is most likely to be diagnosed with which disease process? 1. Addisons disease 2. Cretinism 3. Cushings syndrome 4. Obesity

3. Cushings syndrome Even though the client has gained weight (obesity), the distribution of that weight is characteristic for the disease process of Cushings syndrome. Cretinism and Addisons disease do not exhibit those symptoms.

The parents of a child diagnosed with cystic fibrosis ask the nurse how their child developed the disease. Which of the following should the nurse explain to these parents? 1. Cystic fibrosis is a disease that has an extra chromosome 2. Cystic fibrosis is an X-linked disorder 3. Cystic fibrosis is passed on by a defective gene from both parents 4. Cystic fibrosis is passed on by one defective gene from one parent

3. Cystic fibrosis is passed on by a defective gene from both parents Cystic fibrosis is an inherited, autosomal recessive disease that is passed on by a defective gene from both parents and not one parent. This disease does not occur because of an extra chromosome. This disease is not an X-linked disorder. Cystic fibrosis is a chronic, progressive, and frequently fatal disease of the bodys exocrine mucus-producing glands that primarily affects the respiratory, digestive, intestinal systems, and the pancreas.

A nurse working on a general medical-surgical floor is discussing the clinical manifestations of pulmonary arterial hypertension (PAH) with a recent nursing graduate. What is the main symptom of PAH that the nurse would explain? 1. Chest pain 2. Fatigue 3. Dyspnea 4. Hemoptysis

3. Dyspnea Dyspnea, the main symptom of both types of PAH, first occurs with exertion and eventually at rest. Other signs and symptoms include chest pain, weakness, fatigue, syncope, occasional hemoptysis, and signs of right-sided heart failure.

The nurse is instructing the mother of a client recovering from a tonsillectomy. Which of the following should the nurse instruct the mother to report? 1. Difficulty swallowing 2. Difficulty talking 3. Excessive swallowing 4. Pain

3. Excessive swallowing Excessive swallowing is a sign of bleeding and should be reported. Pain and difficulty talking and swallowing are expected

A client is prescribed meglitinide as oral treatment for type 2 diabetes mellitus. Which of the following should the nurse instruct as a possible side effect of this medication? 1. Diarrhea 2. Constipation 3. Flatulence 4. Hunger

3. Flatulence The most common side effect of meglitinide is flatulence, which can cause the client minor discomfort. The nurse should instruct the client regarding this side effect. Meglitinide does not cause diarrhea, constipation, or hunger

A male client is diagnosed with hyperprolactinemia. The nurse realizes that which of the following clinical manifestations occurs less frequently in men? 1. A decrease in testosterone 2. Erectile dysfunction 3. Gynecomastia 4. Infertility

3. Gynecomastia In men, hyperprolactinemia causes a decrease in testosterone secondary to an inhibition of gonadotropin secretion, leading to decreased facial and body hair, erectile dysfunction, decreased libido, small testicles, and infertility. Gynecomastia occurs less frequently in men.

The nurse is creating a plan of care for a client scheduled for surgery. The nurse should include which activity in the nursing care plan for the client on the day of surgery? 1. Avoid oral hygiene and rinsing with mouthwash. 2. Verify that the client has not eaten for the last 24 hours. 3. Have the client void immediately before going into surgery. 4. Report immediately any slight increase in blood pressure or pulse.

3. Have the client void immediately before going into surgery The nurse would assist the client to void immediately before surgery so that the bladder will be empty. Oral hygiene is allowed, but the client should not swallow any water. The client usually has a restriction of food and fluids for 6 to 8 hours (or longer as prescribed) before surgery instead of 24 hours. A slight increase in blood pressure and pulse is common during the preoperative period and is usually the result of anxiety. Focus on the subject, preoperative care measures. Think about the measures that may be helpful and promote comfort. Oral hygiene should be administered, since it may make the client feel more comfortable. A client should be nothing by mouth (NPO) for 6 to 8 hours before surgery rather than 24 hours. A slight increase in blood pressure or pulse is insignificant in this situation.

A client is being admitted with the diagnosis of asthma. To facilitate breathing, in what position would the nurse place the client? 1. Lateral 2. Prone 3. High-Fowlers 4. Supine

3. High-Fowlers Only the high-Fowlers position facilitates breathing. The other positions could make breathing more difficult

The nurse is assessing a client experiencing manifestations of cor pulmonale. Which of the following will the nurse most likely assess in this client? 1. Low blood pressure 2. Low heart rate 3. Hoarseness 4. Lumbar pain

3. Hoarseness Manifestations of cor pulmonale include hoarseness, chest pain, distended neck veins, liver enlargement, peripheral edema, abnormal heart sounds. Low blood pressure, low heart rate, and lumbar pain are not manifestations of cor pulmonale.

A client is diagnosed with chronic lymphocytic thyroiditis. The nurse should instruct the client regarding signs and symptoms of which of the following? 1. Type 2 diabetes mellitus 2. Heart failure 3. Hypothyroidism 4. Renal failure

3. Hypothyroidism The client diagnosed with chronic lymphocytic thyroiditis will most often progress to hypothyroidism, which is permanent 95% of the time. The nurse should instruct the client regarding signs and symptoms of hypothyroidism. Chronic lymphocytic thyroiditis will not cause type 2 diabetes mellitus, heart failure, or renal failure

When discussing exercise with a client diagnosed with type 2 diabetes mellitus, the client is correct in stating: 1. I will exercise when I can 2. I will exercise once a week for 30 minutes 3. I will try to exercise every day 4. I should exercise for at least 60 minutes when I exercise

3. I will try to exercise every day Clients should work toward a goal of 30 minutes of exercise daily. The intensity of exercise should allow for both breathing and talking with ease during the exercise. The other statements are incorrect and would indicate that the client needs additional instruction regarding exercise

A client is admitted to a hospital with a diagnosis of diabetic ketoacidosis (DKA). The initial blood glucose level is 950 mg/dL (52.9 mmol/L). A continuous intravenous (IV) infusion of short-acting insulin is initiated, along with IV rehydration with normal saline. The serum glucose level is now decreased to 240 mg/dL (13.3 mmol/L). The nurse would next prepare to administer which medication? 1. An ampule of 50% dextrose 2. NPH insulin subcutaneously 3. IV fluids containing dextrose 4. Phenytoin for the prevention of seizures

3. IV fluids containing dextrose Emergency management of DKA focuses on correcting fluid and electrolyte imbalances and normalizing the serum glucose level. If the corrections occur too quickly, serious consequences, including hypoglycemia and cerebral edema, can occur. During management of DKA, when the blood glucose level falls to 250 to 300 mg/dL (13.9 to 16.7 mmol/L), the IV infusion rate is reduced and a dextrose solution is added to maintain a blood glucose level of about 250 mg/dL (13.9 mmol/L), or until the client recovers from ketosis. Fifty percent dextrose is used to treat hypoglycemia. NPH insulin is not used to treat DKA. Phenytoin is not a usual treatment measure for DKA. Note the strategic word, next. Focus on the subject, management of DKA. Eliminate option 2 first, knowing that short-duration (rapid-acting) insulin is used in the management of DKA. Eliminate option 1 next, knowing that this is the treatment for hypoglycemia. Note the words the serum glucose level is now decreased to 240 mg/dL (13.3 mmol/L). This should indicate that the IV solution containing dextrose is the next step in the management of care.

A client has been diagnosed with chronic obstructive pulmonary disease. Which of the following nursing diagnoses would be the most important at this time? 1. Activity intolerance 2. Anxiety 3. Impaired gas exchange 4. Nutrition, imbalance

3. Impaired gas exchange Airway and breathing are always a top priority for a client. Once gas exchange is ensured for the client, the other diagnoses of activity intolerance and nutrition imbalance can be addressed. Anxiety would be addressed last for this client.

The nurse coaches a postoperative client to utilize a breathing device that prevents the complication of atelectasis. This device would be a(n): 1. IPPB 2. Blow bottles 3. Incentive spirometer 4. Postural drainage

3. Incentive spirometer An incentive spirometer assists the patient with deep breathing exercises that can help prevent atelectasis. A client would not use an intermittent positive pressure breathing device without the presence of a nurse and/or respiratory therapist. Blow bottles are not a medical device used to prevent atelectasis. Postural drainage is a technique used to drain secretions from the lung lobes.

A client has been smoking for the last 40 years and has a history of emphysema. Which of the following findings would the nurse not expect to find? 1. Decreased forced vital capacity (FVC) 2. Increased anterior-posterior chest diameter 3. Increased forced expiratory volume (FEV1) 4. Pursed lip breathing

3. Increased forced expiratory volume (FEV1) The FEV1 does not increase; it decreases. The FVC does decrease, and the client can exhibit increased anterior-posterior chest diameter and pursed lip breathing.

A client is brought to the emergency department in an unresponsive state, and a diagnosis of hyperosmolar hyperglycemic syndrome is made. The nurse would immediately prepare to initiate which anticipated primary health care provider's prescription? 1. Endotracheal intubation 2. 100 units of NPH insulin 3. Intravenous infusion of normal saline 4. Intravenous infusion of sodium bicarbonate

3. Intravenous infusion of normal saline The primary goal of treatment in hyperosmolar hyperglycemic syndrome (HHS) is to rehydrate the client to restore fluid volume and to correct electrolyte deficiency. Intravenous (IV) fluid replacement is similar to that administered in diabetic ketoacidosis (DKA) and begins with IV infusion of normal saline. Regular insulin, not NPH insulin, would be administered. The use of sodium bicarbonate to correct acidosis is avoided because it can precipitate a further drop in serum potassium levels. Intubation and mechanical ventilation are not required to treat HHS. Focus on the subject, treatment of HHS, and note the strategic word, immediately. If you can recall the treatment for DKA, you will be able to answer this question easily. Treatment for HHS is similar to the treatment for DKA and begins with rehydration.

The finding of normal breath sounds on the right chest and diminished, distant breath sounds on the left chest of a newly intubated patient is probably due to which of the following? 1. A left pneumothorax 2. A right hemothorax 3. Intubation of the right mainstem bronchus 4. A malfunctioning mechanical ventilator

3. Intubation of the right mainstem bronchus The right mainstem is wider, shorter (about 2.5 cm long), and more vertical in direction than the left main bronchus; therefore, accidental placement in the right lung is a potential complication. It is imperative that after intubation, the nurse auscultate for the presence of bilateral breath sounds.

A client is instructed to rotate the sites of insulin injections because it will help prevent: 1. A decrease in absorption 2. An allergic reaction 3. Lipodystrophy 4. Skin breakdown

3. Lipodystrophy The rotation of sites is used to prevent lipodystrophy, a localized complication of insulin administration characterized by changes in the subcutaneous fat at the site of the injection. The other options are not why site rotation is used

Which of the following should the nurse instruct a client recovering from a tonsillectomy? 1. Drink milk to promote healing 2. Gargle with salt water 3. Maintain good hydration 4. Use a straw to drink

3. Maintain good hydration Drinking milk does not promote healing and may encourage production of mucus. Gargling and drinking with a straw may disrupt the clot at the operative site and cause bleeding. Maintaining good hydration and eating soft foods are encouraged.

The nurse is preparing short-acting and long-acting insulin for administration to a client. The purpose for the clients being prescribed these types of insulin would be to: 1. Make it easier for the client to self-administer the insulin 2. Reduce the clients appetite 3. Mimic the bodys own insulin pattern 4. Help reduce the clients body weight

3. Mimic the bodys own insulin pattern NPH insulin is usually given twice daily and is mixed with regular insulin to mimic the bodys own insulin pattern. Mixing two insulins is not done to make it easier for the client to administer the insulin, to reduce the clients appetite, or to help reduce the clients body weight

A client tells the nurse that she experiences a stuffy nose, nasal pain, and postnasal drip every time she works in her companys office. Which of the following types of allergic rhinitis is this client most likely experiencing? 1. Infectious 2. Perennial 3. Occupational 4. Seasonal

3. Occupational Occupational allergic rhinitis occurs from airborne substances in the workplace. Seasonal allergic rhinitis occurs during a specific time of the year. Perennial allergic rhinitis occurs in response to exposure to environmental allergens that can occur throughout the year. Infectious rhinitis is a nonallergic type of rhinitis

The nurse is assessing a patient with a blunt chest trauma due to an MVA. What finding would be indicative of a flail chest? 1. Hypertension 2. Metabolic alkalosis 3. Paradoxical chest movement 4. Respiratory alkalosis

3. Paradoxical chest movement . During inspiration, as the chest expands, the detached part of the rib segment (flail segment) moves in a paradoxical manner in that it is pulled inward during inspiration, reducing the amount of air that can be drawn into the lungs. On expiration, because the intrathoracic pressure exceeds atmospheric pressure, the flail segment bulges outward, impairing the patient's ability to exhale. Hypotension, inadequate tissue perfusion, and metabolic acidosis often follow as the paradoxical motion of the mediastinum decreases cardiac output. Metabolic alkalosis and respiratory alkalosis would not be an assessment finding that correlates with a flail chest

A client with a chest injury has suffered flail chest. The nurse assesses the client for which most distinctive sign of flail chest? 1. Cyanosis 2. Hypotension 3. Paradoxical chest movement 4. Dyspnea, especially on exhalation

3. Paradoxical chest movement Flail chest results from multiple rib fractures. This results in a "floating" section of ribs. Because this section is unattached to the rest of the bony rib cage, this segment results in paradoxical chest movement. This means that the force of inspiration pulls the fractured segment inward, while the rest of the chest expands. Similarly, during exhalation, the segment balloons outward while the rest of the chest moves inward. This is a characteristic sign of flail chest. Note the strategic word, most. Cyanosis and hypotension occur with many different disorders, so eliminate options 1 and 2 first. From the remaining options, choose paradoxical chest movement over dyspnea on exhalation by remembering that a flail chest has broken rib segments that move independently of the rest of the rib cage.

A client is diagnosed with a low serum calcium level. The nurse realizes that which hormone is released when serum calcium levels are low? 1. Calcitonin 2. Cortisol 3. Parathyroid hormone 4. Thyroxine

3. Parathyroid hormone Parathyroid hormone is secreted when serum calcium levels are low. Calcitonin is released when serum calcium levels are high. Cortisol and thyroxine are not related to calcium

The nurse has a positive PPD during the last testing cycle for tuberculosis. Which of the following is indicated for this nurse? 1. Nothing 2. Chest x-rays every 2 months 3. Pharmacological treatment 4. Admission for inpatient treatment

3. Pharmacological treatment Latent tuberculosis infection occurs when a person exposed to the mycobacterium has a positive PPD test. This person is without an active clinical picture and has a 10% chance of developing TB if preventive pharmacological treatment is not initiated. The nurse needs pharmacological treatment. Doing nothing could result in active disease. The nurse does not need chest x-rays every 2 months or admission for inpatient treatment.

The nurse is planning to teach a postoperative client about discharge medication. Which of these nursing interventions would best assist the client in learning? 1. Withhold any pain medication so that the client can concentrate better 2. Schedule the teaching after physical therapy so the client will be relaxed 3. Place the client in a comfortable position and have the patient use the bathroom 4. Plan the teaching at night right before bed so that the client can sleep on the new information given

3. Place the client in a comfortable position and have the patient use the bathroom Placing the client in a comfortable position and having him use the bathroom will allow him to concentrate on the learning to take place. The client will not be able to concentrate on the instructions if he is in pain. The client may be tired after physical therapy and would not want to engage in instruction at this time. Waiting until night to conduct instruction is also not a good time considering the client may be fatigued from activities throughout the day and needs to rest.

A pregnant client is receiving treatment for hyperthyroidism. Which of the following medications would the nurse expect to see? 1. Levothryoxine 2. Methimazole 3. Propylthiouracil 4. Radioactive iodine

3. Propylthiouracil Propylthiouracil (PTU) is the drug of choice for treating hyperthyroidism in a pregnant or breastfeeding client. Radioactive iodine and methimazole are treatments for nonpregnant clients with hyperthyroidism. Levothyroxine is used to treat hypothyroidism.

The nurse is planning care for a client diagnosed with Graves disease. Which of the following nursing interventions would be appropriate for this clients care? 1. Administer a stool softener 2. Provide extra blankets 3. Provide frequent meals 4. Restrict the caloric intake

3. Provide frequent meals Nursing interventions for Graves disease (hyperthyroidism) include offering frequent, highcalorie meals; medicating for diarrhea; providing a fan or decreasing the temperature on the air conditioner; and taking daily weight measurements. The client does not need a stool softener. The client does not need extra blankets. The clients metabolic rate is increased, and she should not have a restriction on caloric intake

A client has a productive cough that produces green sputum with a musty odor. The nurse realizes that the client may be experiencing: 1. emphysema 2. pneumococcal pneumonia 3. Pseudomonas infection 4. Pulmonary edema

3. Pseudomonas infection A client with a Pseudomonas infection can have a cough that produces green sputum with a musty odor. The sputum from emphysema is gray-white and mucoid. The sputum from pneumococcal pneumonia and pulmonary edema are rust colored and pink, frothy, respectively.

The nurse is instructing a postoperative client regarding signs of complications. Which of the following should be included in these instructions? 1. Notify the physician with a body temperature greater than 99F 2. Expect the pain level to increase 3. Report a change in drainage or increase in bleeding 4. Dizziness and fainting is an expected side effect of anesthesia

3. Report a change in drainage or increase in bleeding Signs and symptoms of postoperative complications include fever, usually greater than 100 or 101F; sudden change in pain; change in drainage or bleeding; dizziness and fainting. The client should not be instructed to notify the physician with a body temperature of 99F. Pain level should not increase once discharged. Dizziness and fainting should be reported immediately

A client diagnosed with viral rhinitis tells the nurse that she has been using a decongestant nasal spray for several weeks and the symptoms are getting worse. Which of the following does the nurse suspect is occurring with this client? 1. Developing pneumonia 2. Subacute rhinitis 3. Rhinitis medicamentosa 4. Chronic otitis media

3. Rhinitis medicamentosa Rhinitis medicamentosa can occur with overuse of decongestant nasal sprays, and it leads to rebound nasal congestion that is often worse that the original nasal congestion. The use of nasal sprays does not cause pneumonia, subacute rhinitis, or chronic otitis media.

A client diagnosed with asthma is having an acute episode at home. Which of the following medications should the client be instructed not to use during this episode? 1. Albuterol 2. Proventil 3. Serevent 4. Ventolin

3. Serevent Serevent is a long-acting agent and is not to be used as rescue medication during acute episodes. Albuterol (also sold under the brand names Proventil and Ventolin) is a short-acting agent used as a rescue medication.

A nurse is filling the role of circulator during a surgical procedure. Which of the following will this nurse do to provide care to the client during the case? 1. Maintain the sterile field 2. Assist the surgeon 3. Serve as the client advocate 4. Assist with the administration of anesthesia

3. Serve as the client advocate The circulating nurse serves as the client advocate while the client is least able to care for himself. Maintaining the sterile field is a responsibility of the scrub nurse. Assisting the surgeon is an activity of the registered nurse first assistant. Assisting with the administration of anesthesia is an activity of the nurse anesthetist.

The nurse performs an admission assessment on a client with a diagnosis of tuberculosis. The nurse should check the results of which diagnostic test that will confirm this diagnosis? 1. Chest x-ray 2. Bronchoscopy 3. Sputum culture 4. Tuberculin skin test

3. Sputum culture Tuberculosis is definitively diagnosed through culture and isolation of Mycobacterium tuberculosis. A presumptive diagnosis is made based on a tuberculin skin test, a sputum smear that is positive for acid-fast bacteria, a chest x-ray, and histological evidence of granulomatous disease on biopsy. Focus on the subject, confirming the diagnosis of tuberculosis. Confirmation is made by identifying the bacteria, M. tuberculosis.

The nurse performs a physical assessment on a client with type 2 diabetes mellitus. Findings include a fasting blood glucose level of 70 mg/dL (3.9 mmol/L), temperature of 101° F (38.3° C), pulse of 82 beats per minute, respirations of 20 breaths per minute, and blood pressure of 118/68 mm Hg. Which finding would be the priority concern to the nurse? 1. Pulse 2. Respiration 3. Temperature 4. Blood pressure

3. Temperature In the client with type 2 diabetes mellitus, an elevated temperature may indicate infection. Infection is a leading cause of hyperosmolar hyperglycemic syndrome in the client with type 2 diabetes mellitus. The other findings are within normal limits Note the strategic word, priority. Use knowledge of the normal values of vital signs to direct you to the correct option. The client's temperature is the only abnormal value. Remember that an elevated temperature can indicate an infectious process that can lead to complications in the client with diabetes mellitus.

A client experiencing chronic pain asks the nurse why she is not prescribed Demerol like she received when she had a total knee replacement. Which of the following should the nurse respond to this client? 1. You don't need something that strong 2. That medication does not exist anymore 3. That medication does not last very long 4. It can cause you have high blood pressure

3. That medication does not last very long Meperidine is no longer a major drug for acute or chronic pain due to its short analgesic duration of 2 to 3 hours and the potential for accumulative toxic effects of its metabolite, normeperidine. The best response for the nurse to make to the client would be that medication does not last very long. The other responses are inaccurate.

While assessing a client with Cushing syndrome, the nurse should expect high blood glucose reading due to increased secretion of which of the following? 1. The thyroid gland 2. The parathyroid glands 3. The adrenal glands 4. The pituitary gland

3. The adrenal glands In Cushing's syndrome, increased secretion of glucocorticoid hormones from the adrenal glands causes increased levels of blood glucose

A client is prescribed a diuretic for treatment of pulmonary hypertension. Which of the following should the nurse instruct the client regarding this medication? 1. This medication expands the blood vessels 2. This medication causes smooth muscle relaxation to reduce pulmonary engorgement 3. This medication reduces the amount of water in the body 4. This medication keeps the blood from clotting

3. This medication reduces the amount of water in the body Diuretics in the treatment of pulmonary hypertension are used to reduce the amount of water in the body. Vasodilators expand the blood vessels. Sildenafil causes smooth muscle relaxation to reduce pulmonary engorgement. Anticoagulants keep the blood from clotting

A client with a history of malingering pain tells the nurse that he needs a prescription for pain medication. Which of the following should the nurse do first to assist this client? 1. Ask the physician for a pain medication prescription for the client 2. Remind the client that he does not have pain but just wants the medication 3. Thoroughly assess the client for pain 4. Suggest the client seek counseling for his pain medication-seeking behavior

3. Thoroughly assess the client for pain Pain of a psychological origin is when an individual seeks treatment for pain when no actual pain exists. This is also referred to as malingering or pretending pain. The nurse should not assume that the pain does not exist but rather should conduct a thorough pain assessment to rule out an actual physiological problem. The nurse should not immediately ask the physician for pain medication. The nurse should not remind the client that he does not have pain but just wants the medication. The nurse should also not suggest the client seek counseling for pain medication-seeking behavior

The nurse is caring for a client who has completed pulmonary function testing. Which of the following indicates the amount of air inhaled or exhaled with each breath during normal breathing? 1. Expiratory reserve volume 2. Minute volume 3. Tidal volume 4. Vital capacity

3. Tidal volume Tidal volume is the amount of air inhaled or exhaled with each breath during normal breathing. The expiratory reserve volume is the maximum amount of air exhaled forcefully after a normal exhalation. Minute volume is the amount of air breathed per minute. Vital capacity is the maximum amount of air exhaled after maximum inspiration.

A client is diagnosed with the type of diabetes in which the plasma beta cells fail to respond to insulin. Which type of diabetes is this client experiencing? 1. Gestational diabetes 2. Impaired glucose tolerance 3. Type 1 diabetes mellitus 4. Type 2 diabetes mellitus

3. Type 1 diabetes mellitus Type 1 diabetes mellitus results from a defect or failure of the beta cells of the pancreas. The loss of beta cells causes a lack of insulin. The other options produce insulin

The nurse is assessing a client for acromegaly at the clinic. Besides asking about changes in shoe size and facial features, the nurse should also inquire about changes in which of the following? 1. Hearing 2. Bowel habits 3. Vision 4. Taste of foods

3. Vision Oversecretion of the growth hormone from the anterior pituitary results in acromegaly. The pituitary gland may be enlarged and causing a pressure on the optic nerve in the brain, thus changes in vision may occur

The nurse realizes that an adequate amount of which vitamin must be present for parathyroid hormone to be fully effective? 1. Vitamin A 2. Vitamin C 3. Vitamin D 4. Vitamin E

3. Vitamin D Adequate vitamin D is necessary for absorption of calcium into the bloodstream. Vitamins A, C, and E do not have a role in calcium regulation

A 16-year-old client is being prescribed a medication to treat acute sinusitis. The nurse realizes that this client should not be prescribed: 1. amoxicillin 2. cefuroxime 3. ciprofloxacin 4. erythromycin

3. ciprofloxacin Quinolones such as ciprofloxacin (Cipro) and levofloxacin (Levaquin) are contraindicated in children younger than 17 years of age.

The results of a clients thyroid scan showed black and gray areas. The nurse realizes this finding is consistent with: 1. malignancies 2. elevated phosphorus levels 3. hyperactivity 4. renal disease

3. hyperactivity Hyperactive areas on the thyroid scan will appear as black or gray regions or hot spots. White areas or cold spots are indicative of malignancies. Black and gray areas on the thyroid scan are not indicative of elevated phosphorus levels or renal disease.

The nurse should instruct the postoperative client that antiembolic stockings are used to: 1. keep the legs warm 2. serve as a nonslip slipper 3. promote venous return 4. make it easier to ambulate after surgery

3. promote venous return Surgery may result in swelling that could impede blood return. Antiembolic stockings will aid in blood return and reduce lower extremity edema postoperatively. These stockings are not used to keep the legs warm, serve as a nonslip slipper, nor make it easier to ambulate after surgery.

A client diagnosed with hypertension is experiencing allergic rhinitis. The nurse realizes that the medication that would not be indicated for this client would be: 1. loratadine 2. Montelukast 3. pseudoephedrine 4. zafirlukast

3. pseudoephedrine Pseudoephedrine can be contraindicated for the patient with hypertension. Loratadine, montelukast, and zafirlukast should be used cautiously for patients with hepatic impairment.

The nurse suspects a client is experiencing chronic obstructive pulmonary disease when which of the following is assessed? (Select all that apply.) 1. Peripheral edema 2. Jugular vein distention 3. High blood pressure 4. Dyspnea on exertion 5. Sputum production 6. Cough

4, 5, 6 Dyspnea on exertion, sputum production, cough Chronic obstructive pulmonary disease is characterized by a history of three primary symptoms: 1) cough, 2) sputum production, and 3) dyspnea on exertion. Peripheral edema, jugular vein distention, and high blood pressure are not symptoms of chronic obstructive pulmonary disease.

The nurse has conducted discharge teaching with a client diagnosed with tuberculosis who has been receiving medication for 2 weeks. The nurse determines that the client has understood the information if the client makes which statement? 1. "I need to continue medication therapy for 1 month." 2. "I can't shop at the mall for the next 6 months." 3. "I can return to work if a sputum culture comes back negative." 4. "I should not be contagious after 2 to 3 weeks of medication therapy."

4. "I should not be contagious after 2 to 3 weeks of medication therapy" The client is continued on medication therapy for up to 12 months, depending on the situation. The client generally is considered noncontagious after 2 to 3 weeks of medication therapy. The client is instructed to wear a mask if there will be exposure to crowds until the medication is effective in preventing transmission. The client is allowed to return to work when the results of 3 sputum cultures are negative. Focus on the subject, client understanding of medication therapy. Knowing that the medication therapy lasts for up to 12 months helps you eliminate option 1 first. Knowing that 3 sputum cultures must be negative helps you eliminate option 3 next. From the remaining options, recalling that the client is not contagious after 2 to 3 weeks of therapy will direct you to the correct option.

The nurse provides instructions to a client newly diagnosed with type 1 diabetes mellitus. The nurse recognizes accurate understanding of measures to prevent diabetic ketoacidosis when the client makes which statement? 1. "I will stop taking my insulin if I'm too sick to eat." 2. "I will decrease my insulin dose during times of illness." 3. "I will adjust my insulin dose according to the level of glucose in my urine." 4. "I will notify my primary health care provider (PHCP) if my blood glucose level is higher than 250 mg/dL (13.9 mmol/L)."

4. "I will notify my primary health care provider (PHCP) if my blood glucose level is higher than 250 mg/dL (13.9 mmol/L)" During illness, the client with type 1 diabetes mellitus is at increased risk of diabetic ketoacidosis, due to hyperglycemia associated with the stress response and due to a typically decreased caloric intake. As part of sick day management, the client with diabetes should monitor blood glucose levels and should notify the PHCP if the level is higher than 250 mg/dL (13.9 mmol/L). Insulin should never be stopped. In fact, insulin may need to be increased during times of illness. Doses should not be adjusted without the PHCP's advice and are usually adjusted on the basis of blood glucose levels, not urinary glucose readings. Use general medication guidelines to answer the question. Note that options 1, 2, and 3 are comparable or alike and all relate to adjustment of insulin doses.

The nurse anticipates that during the initial treatment of diabetic ketoacidosis, the provider will order which solution? 1. D5W 2. D5.45% saline 3. Lactated Ringer solution 4. 0.9% saline

4. 0.9% saline Normal saline replaces fluids without adding glucose or electrolytes. A and B contain 5% dextrose and C contains potassium as well as other electrolytes.

A client diagnosed with type 1 diabetes mellitus administers a dose of NPH insulin at 7:00 a.m. At which of the following times would this client exhibit hypoglycemia? 1. 0800 2. 0900 3. 1000 4. 1400

4. 1400 NPH insulin peaks in 4 to 12 hours. During these hours, the client may experience a hypoglycemic episode. The other choices identify times that are before the peak times for the insulin.

The spouse of a client diagnosed with tuberculosis is to begin isoniazid prophylactic therapy. Which of the following should the nurse instruct the spouse regarding length of time to take this medication? The medication should be taken for: 1. 10 to 24 days 2. 1 to 3 months 3. 4 to 7 months 4. 6 to 12 months

4. 6 to 12 months Isoniazid therapy lasts 6 to 12 months. Taking the medication less than 6 months can be ineffective. The spouse should not be instructed to take the medication for 10 to 24 hours, 1 to 3 months, or 4 to 7 months.

During a surgical procedure, the clients body temperature spikes to a dangerous level. Which of the following will be done to help this client? 1. Reduce the flow of the anesthetic agent 2. Provide 50% oxygen 3. Stop the surgery for cardiac dysrhythmias 4. Administer a Dantrolene infusion

4. Administer a Dantrolene infusion Malignant hyperthermia is a medical emergency. The anesthetic agent should be stopped immediately and the client should be hyperventilated with 100% oxygen. The surgery should be stopped if it is an elective case. Dantrolene should be provided.

The nurse is caring for a client admitted to the emergency department with diabetic ketoacidosis (DKA). In the acute phase, the nurse plans for which priority intervention? 1. Correct the acidosis. 2. Administer 5% dextrose intravenously. 3. Apply a monitor for an electrocardiogram. 4. Administer short-duration insulin intravenously.

4. Administer short-duration insulin intravenously Lack of insulin (absolute or relative) is the primary cause of DKA. Treatment consists of insulin administration (short- or rapid-acting), intravenous fluid administration (normal saline initially, not 5% dextrose), and potassium replacement, followed by correcting acidosis. Cardiac monitoring is important due to alterations in potassium levels associated with DKA and its treatment, but applying an electrocardiogram monitor is not the priority action. Focus on the client's diagnosis. Note the strategic word, priority. Remember that in DKA, the initial treatment is short- or rapid-acting insulin. Normal saline is administered initially; therefore, option 2 is incorrect. Options 1 and 3 may be components of the treatment plan but are not the priority.

A nurse is caring for a patient diagnosed with lung cancer who has a chest tube. The chest tube has continuous bubbling in the water-seal chamber. What does the nurse understand that this indicates? 1. Tidaling 2. The tube in the mediastinum 3. A properly functioning system 4. An air leak in the system

4. An air leak in the system Intermittent bubbling in the water seal chamber is normal, but continuous bubbling can indicate an air leak. Fluctuation of the water level in the water seal is called tidaling. Bubbling and tidaling do not occur when the tube is placed in the mediastinal space.

Which of the following nursing interventions would be appropriate after a wound evisceration? 1. Place the client in high-Fowlers position 2. Give the client fluids to prevent shock 3. Push the organs back inside and tape up the wound 4. Apply a sterile saline-soaked dressing and cover

4. Apply a sterile saline-soaked dressing and cover The nurse is to cover the wound with a sterile saline-soaked dressing and maintain it until the client is taken to surgery. High-Fowlers position will not help with wound evisceration. Providing fluids would be contraindicated since the client will be returning to surgery. The nurse should not manipulate the exposed organs.

A patient is receiving thrombolytic therapy for treatment of a pulmonary emboli. For what side effect should the nurse must monitor the patient? 1. Chest pain 2. Rash 3. Hyperthermia 4. Bleeding

4. Bleeding Thrombolytic therapy dissolves the thrombi or emboli more quickly and restores more normal hemodynamic functioning of the pulmonary circulation, thereby reducing pulmonary hypertension and improving perfusion, oxygenation, and cardiac output. However, bleeding is a significant side effect. Chest pain, a rash, and elevated temperature are not therapy-specific side effects associated with the use of thrombolytics

The nurse is implementing the five Cs of pain management for a client. Which of the following is included in this intervention? 1. Caring for the client in a holistic manner 2. Creating a calm environment 3. Comparing the degree of pain reported with previous episodes 4. Continuously assessing the clients pain

4. Continuously assessing the client's pain The five Cs of pain management include comprehensive assessment, consistent use of assessment tools, continuous reassessment, customize the plan of care, and collaborate with other health care providers to plan pain management. The other choices are not included in the five Cs of pain management.

A client asks the nurse if there is an antihistamine that does not cause drowsiness. Which of the following medications would this client most likely prefer to treat allergic rhinitis? 1. Diphenhydramine 2. Chlorpheniramine maleate 3. Clemastine 4. Fexofenadine

4. Fexofenadine Fexofenadine (Allegra) is a second-generation antihistamine, and second-generation antihistamines exhibit less sedation than first-generation medications such as diphenhydramine, chlorpheniramine maleate, and clemastine

A child is diagnosed with severe allergic rhinitis. Which of the following manifestations would the nurse most likely assess in this client? 1. Edematous neck glands 2. Reduced hearing 3. Pruritis 4. Frequent wiping of the nose with the palm of the hand

4. Frequent wiping of the nose with the palm of the hand Frequent wiping of the nose with the palm of the hand is one symptom seen in the client diagnosed with severe allergic rhinitis. Edematous neck glands, reduced hearing, and pruritis are not manifestations of severe allergic rhinitis

A 47-year-old woman presents to her primary care provider complaining of bone pain. Routine laboratory studies reveal a high serum calcium of 12.0 mg/dL and increased PTH levels. Which of the following is the most likely diagnosis? 1. Graves disease 2. Cushing disease 3. Addison disease 4. Hyperparathyroidism

4. Hyperparathyroidism Hyperparathyroidism is characterized by having excess parathormone (PTH), leading to a markedly increased level of serum calcium

A patient is admitted to the ICU after falling from a roof and sustaining fractures of the first three ribs on the right side. The patient is dyspneic, and crepitus (subcutaneous emphysema) can be palpated. Auscultation reveals decreased breath sounds on the right. The chest x-ray reveals a pneumothorax. Upon percussion, what does the nurse expect to hear? 1. Dullness on the right 2. Resonance on the right 3. Tympany on the right 4. Hyperresonance on the right

4. Hyperresonance on the right Hyperresonance is expected with a pneumothorax due to accumulation of air in the pleural space

A client should not be prescribed tolazamide if the client is sensitive to: 1. Penicillin 2. Shellfish 3. Strawberries 4. Hypoglycemia

4. Hypoglycemia Tolazamide is a first generation sulfonylurea, and can cause a high incidence of hypoglycemia. This medication is used sparingly in the United States today because there are second-generation sulfonylureas that are more effective. Tolazamide can be used if the client is sensitive to penicillin, shellfish, or strawberries.

A client has a slight shift to the left on the oxygen-hemoglobin dissociation curve. Which of the following assessment findings will support this curve configuration? 1. Arterial pH less than 7.35 2. Increased levels of 2,3-diphosphoglycerate 3. Hyperthermia 4. Hypothermia

4. Hypothermia Factors that cause increased affinity of oxygen for hemoglobin will shift the oxyhemoglobin dissociation curve to the left. These factors include alkalemia and hypothermia. Arterial pH less than 7.35, increased leaves of 2,3-diphosphoglycerate, and hyperthermia indicate a shift to the right, not the left

The nurse is assessing a client with multiple trauma who is at risk for developing acute respiratory distress syndrome. The nurse should assess for which earliest sign of acute respiratory distress syndrome? 1. Bilateral wheezing 2. Inspiratory crackles 3. Intercostal retractions 4. Increased respiratory rate

4. Increased respiratory rate The earliest detectable sign of acute respiratory distress syndrome is an increased respiratory rate, which can begin from 1 to 96 hours after the initial insult to the body. This is followed by increasing dyspnea, air hunger, retraction of accessory muscles, and cyanosis. Breath sounds may be clear or consist of fine inspiratory crackles or diffuse coarse crackles. Note the strategic word, earliest. Eliminate option 3 first, because intercostal retraction is a later sign of respiratory distress. Of the remaining options, recall that adventitious breath sounds (options 1 and 2) would occur later than an increased respiratory rate.

The nurse is managing an epidural catheter for a patient diagnosed with cancer. The patient has developed a spinal headache. Which intervention should be completed at this time? 1. Increase the dosage of anesthetic. 2. Place in semi-Fowler position. 3. Continue to monitor the patient. 4. Initiate administration of prescribed fluids.

4. Initiate administration of prescribed fluids If a headache develops, the patient should remain flat in bed and should be given large amounts of fluids (provided the medical condition allows), and the health care provider should be notified. It is not appropriate to continue to monitor the patient without notifying the health care provider in this situation.

An external insulin pump is prescribed for a client with diabetes mellitus. When the client asks the nurse about the functioning of the pump, the nurse bases the response on which information about the pump? 1. It is timed to release programmed doses of either short-duration or NPH insulin into the bloodstream at specific intervals. 2. It continuously infuses small amounts of NPH insulin into the bloodstream while regularly monitoring blood glucose levels. 3. It is surgically attached to the pancreas and infuses regular insulin into the pancreas. This releases insulin into the bloodstream. 4. It administers a small continuous dose of short-duration insulin subcutaneously. The client can self-administer an additional bolus dose from the pump before each meal.

4. It administers a small continuous dose of short-duration insulin subcutaneously. The client can self-administer an additional bolus dose from the pump before each meal. An insulin pump provides a small continuous dose of short-duration (rapid- or short-acting) insulin subcutaneously throughout the day and night. The client can self-administer an additional bolus dose from the pump before each meal as needed. Short-duration insulin is used in an insulin pump. An external pump is not attached surgically to the pancreas. An insulin pump provides a small continuous dose of short-duration (rapid- or short-acting) insulin subcutaneously throughout the day and night. The client can self-administer an additional bolus dose from the pump before each meal as needed. Short-duration insulin is used in an insulin pump. An external pump is not attached surgically to the pancreas.

A patient has a chronic cough with thick sputum. He undergoes a bronchoscopy for diagnostic purposes. Following the bronchoscopy, which action by the nurse is most appropriate? 1. Encourage fluid intake to promote elimination of contrast media. 2. Monitor hemoglobin and hematocrit to evaluate blood loss. 3. Check vital signs every 15 minutes for 4 hours. 4. Keep NPO until the gag reflex returns.

4. Keep NPO until the gag reflex returns Preoperative sedation and local anesthesia impair the protective laryngeal reflex and swallowing for several hours after a bronchoscopy. To prevent aspiration before refeeding the patient, the nurse must reassess the return of the gag reflex (pharyngeal reflex; ninth, tenth cranial nerve).

A hospital conducts an outpatient clinic for patients with asthma. The nurse working in the clinic is responsible for teaching patients about their medications. Which is true about inhaled medications? 1. All inhalers must be given with a spacer device 2. Inhaled corticosteroids are used on an as-needed basis for quick relief 3. LABAs (long-acting beta adrenergics) can be used as monotherapy to control asthma 4. LABAs and inhaled corticosteroids are used as controllers in the management of asthma

4. LABAs (long-acting beta adrenergics) and inhaled corticosteroids are used as controllers in the management of asthma Spacers are generally recommended for patients unable to use their inhalers correctly. Additionally, they are useful for those using inhaled corticosteroids by MDI to decrease the risk of oral thrush. The Expert panel Guidelines recommend that LABAs not be used as monotherapy in the long-term treatment of asthma but are used in combination with inhaled corticosteroids.

A client is diagnosed with a low level of triiodothyronine. The nurse realizes that this hormone affects which of the following body functions? 1. Blood glucose regulation 2. Bone growth 3. Calcium regulation 4. Metabolism

4. Metabolism Triiodothyronine (T3) affects the metabolic rate. Bone growth is affected by growth hormone. Calcium regulation is controlled by calcitonin and parathyroid hormone. Blood glucose regulation is controlled by insulin and glucagon.

A client diagnosed with asthma is receiving instructions about the use of albuterol. The client should be aware that albuterol may cause: 1. Bradycardia 2. Drowsiness 3. Nasal congestion 4. Nervousness

4. Nervousness Albuterol causes nervousness, tachycardia, insomnia, dizziness, tremors, hypertension, headache, and irritation to the nasal and throat passages. Albuterol does not cause bradycardia, drowsiness, or nasal congestion

A client with a gastric ulcer is scheduled for surgery. The client cannot sign the operative consent form because of sedation from opioid analgesics that have been administered. The nurse should take which most appropriate action in the care of this client? 1. Obtain a court order for the surgery. 2. Have the charge nurse sign the informed consent immediately. 3. Send the client to surgery without the consent form being signed. 4. Obtain a telephone consent from a family member, following agency policy.

4. Obtain a telephone consent from a family member, following agency policy Every effort should be made to obtain permission from a responsible family member to perform surgery if the client is unable to sign the consent form. A telephone consent must be witnessed by 2 persons who hear the family member's oral consent. The 2 witnesses then sign the consent with the name of the family member, noting that an oral consent was obtained. Consent is not informed if it is obtained from a client who is confused, unconscious, mentally incompetent, or under the influence of sedatives. In an emergency, a client may be unable to sign and family members may not be available. In this situation, a surgeon is permitted legally to perform surgery without consent, but the data in the question do not indicate an emergency. Options 1, 2, and 3 are not appropriate in this situation. Also, agency policies regarding informed consent should always be followed. Note the strategic words, most appropriate. Focus on the data in the question. Eliminate options 1 and 3 first. Option 1 will delay necessary surgery, and option 3 is inappropriate. Option 2 is not an acceptable and legal role of a charge nurse. Select option 4, since it is the only legally acceptable option: to obtain a telephone permission from a family member if it is witnessed by 2 persons.

A client is prescribed medication after recovering from surgery to treat acromegaly. Which of the following medications would the nurse expect to see prescribed? 1. None 2. Cabergoline 1 mg PO twice a week 3. Cortisone acetate 100 mg PO three times a day 4. Octreotide 20 mg IM every 4 weeks

4. Octreotide (Sandostatin) 20 mg IM every 4 weeks Sandostatin is used for residual growth hormone hypersecretion following surgery. Cortone is used to treat adrenocorticotropic dysfunction, and Dostinex is used to treat hyperprolactinemia.

The nurse is providing a medication to reduce the preoperative clients anxiety. Which of the following medications is the nurse most likely providing to the client? 1. Hydrogen ion antagonist 2. Anticholinergic 3. Calcium channel blocker 4. Opioid

4. Opioid Opioids provide analgesia, decrease anxiety, and provide sedation. Calcium channel blockers treat specific heart problems. Hydrogen ion antagonists are used to reduce gastric secretions. Anticholinergics are used to reduce oral and respiratory tract secretions.

The nurse is assessing the respiratory status of a client who has suffered a fractured rib. The nurse should expect to note which finding? 1. Slow, deep respirations 2. Rapid, deep respirations 3. Paradoxical respirations 4. Pain, especially with inspiration

4. Pain, especially with inspiration Rib fractures result from a blunt injury or a fall. Typical signs and symptoms include pain and tenderness localized at the fracture site that is exacerbated by inspiration and palpation, shallow respirations, splinting or guarding the chest protectively to minimize chest movement, and possible bruising at the fracture site. Paradoxical respirations are seen with flail chest. Focus on the subject, findings associated with a rib fracture. Focusing on the anatomical location of the injury will direct you to the correct option.

A client is experiencing the ventilation-perfusion mismatch termed shunting. The nurse realizes that the client most likely is not experiencing which of the following disorders? 1. Hemothorax 2. Intrapulmonary fistulas 3. Pneumothorax 4. Pulmonary embolus

4. Pulmonary embolus Shunting is the portion of the cardiac output that does not exchange with alveolar air. Examples of shunting include hemothorax, pneumothorax, and intrapulmonary fistulas. Pulmonary embolus is the other type of ventilation-perfusion mismatch called dead space

A client is prescribed insulin to be given through an intravenous access line. The nurse realizes that which of the following insulins can be administered intravenously? 1. Glargine 2. Lispro 3. NPH 4. Regular

4. Regular Regular insulin may be given intravenously or subcutaneously. All other insulins are given subcutaneously.

A client, experiencing an acid-base imbalance, demonstrates signs of full compensation within 3 days. The nurse realizes that the full compensation was accomplished by which of the following systems? 1. Extracellular buffer 2. Intracellular buffer 3. Pulmonary 4. Renal

4. Renal The extracellular and intracellular buffer systems act immediately, the pulmonary system acts within 2 to 3 hours, and the renal system responds within 2 to 3 days.

A client is determined to be a candidate for a low-flow oxygen delivery system. Which of the following will the nurse most likely assess in this client? 1. Active bleeding 2. Change in level of consciousness 3. Cardiac arrhythmias 4. Respiratory rate of 16, unlabored breathing

4. Respiratory rate of 16, unlabored breathing Low-flow oxygen systems are used for clients who are clinically stable and have a normal ventilatory pattern such as the client with a respiratory rate of 16 and unlabored breathing. A high-flow oxygen system would be indicated for a client who is not clinically stable such as bleeding, change in level of consciousness, or who is experiencing cardiac arrhythmias.

The nurse provides discharge instructions to a client with pulmonary sarcoidosis. The nurse concludes that the client understands the information if the client indicates to report which early sign of exacerbation? 1. Fever 2. Fatigue 3. Weight loss 4. Shortness of breath

4. Shortness of breath Dry cough and dyspnea are typical early manifestations of pulmonary sarcoidosis. Later manifestations include night sweats, fever, weight loss, and skin nodules. Note the strategic word, early. Because sarcoidosis is a pulmonary problem, eliminate options 1 and 3 first. Select the correct option over option 2 because the shortness of breath (and impaired ventilation) appears first and would cause the fatigue as a secondary symptom.

A client is experiencing epistaxis. Which of the following interventions would the nurse complete? 1. Call the doctor 2. Check laboratory test results 3. Obtain an emesis basin 4. Show the patient how to pinch the nose

4. Show the patient how to pinch the nose The initial intervention for a client with epistaxis is to show the client how to lean forward and pinch the nose against the nasal septum for about 5 to 10 minutes continuously. The other interventions are not necessary at this time

The nurse is instructing a hospitalized client with a diagnosis of emphysema about measures that will enhance the effectiveness of breathing during dyspneic periods. Which position should the nurse instruct the client to assume? 1. Sitting up in bed 2. Side-lying in bed 3. Sitting in a recliner chair 4. Sitting up and leaning on an overbed table

4. Sitting up and leaning on an overbed table Positions that will assist the client with emphysema with breathing include sitting up and leaning on an overbed table, sitting up and resting the elbows on the knees, and standing and leaning against the wall. Eliminate options 1 and 3 first because they are comparable or alike. Next, eliminate option 2 because this position will not enhance breathing.

A client receiving oral medications for the treatment of tuberculosis develops hepatitis. Which of the following medications would be indicated for the client at this time? 1. Ethambutol 2. Isoniazid 3. Rifampin 4. Streptomycin

4. Streptomycin Streptomycin is a medication that can be used until the cause of hepatitis is identified or the liver tissue heals. It is also given for those who have a first-line drug intolerance. First-line drugs are isoniazid (INH), rifampin (RIF), ethambutol (EMB), and pyrazinamide (PZA).

The nurse is conducting preoperative teaching with a client about the use of an incentive spirometer. The nurse should include which piece of information in discussions with the client? 1. Inhale as rapidly as possible. 2. Keep a loose seal between the lips and the mouthpiece. 3. After maximum inspiration, hold the breath for 15 seconds and exhale. 4. The best results are achieved when sitting up or with the head of the bed elevated 45 to 90 degrees.

4. The best results are achieved when sitting up or with the head of the bed elevated 45 to 90 degrees For optimal lung expansion with the incentive spirometer, the client should assume the semi-Fowler's or high-Fowler's position. The mouthpiece should be covered completely and tightly while the client inhales slowly, with a constant flow through the unit. The breath should be held for 5 seconds before exhaling slowly. Focus on the subject, correct use of an incentive spirometer, and visualize the procedure. Note the words rapidly, loose, and 15 seconds in the incorrect options. Options 1, 2, and 3 are incorrect steps regarding incentive spirometer use

Which of the following should the nurse instruct a client when teaching how to self-administer insulin? 1. The insulin bottle must be shaken 2. The long-acting insulin is clear 3. Refrigerated insulin is best for injection 4. The blood glucose level should be checked prior to administration

4. The blood glucose level should be checked prior to administration Insulin bottles should not be shaken but rolled to make sure the precipitate is mixed. The longacting insulin is cloudy. The insulin should be at room temperature for administration, and the blood glucose level should be checked prior to administration

A client states, I dont know why I should quit smoking. It cant improve anything. The nurse responds by informing the client about the decrease in lung cancer rates over time after a person quits smoking. Which of the following is correct? 1. The lung cancer rate corresponds to that of nonsmokers 1 year after quitting smoking 2. The lung cancer rate corresponds to that of nonsmokers 2 years after quitting smoking 3. The lung cancer rate corresponds to that of nonsmokers 5 years after quitting smoking 4. The lung cancer rate corresponds to that of nonsmokers 10 years after quitting smoking

4. The lung cancer rate corresponds to that of nonsmokers 10 years after quitting smoking Ten years after quitting smoking, the clients lung cancer rate will correspond to a nonsmokers rate. After 1 year of no smoking, the risk of coronary heart disease decreases to half that of a smoker. After 2 years of no smoking, the risk of coronary heart disease equals that of a nonsmoker. After 5 years of no smoking, the lung cancer rate drops by half

A client is recovering from a total laryngectomy with the placement of a tracheostomy. The nurse should include which of the following instructions to this client? 1. Clean the tracheostomy tube with soap and water daily 2. Limit protein in the diet 3. Restrict fluids 4. The nasogastric tube will be in for 2 weeks

4. The nasogastric tube will be in for 2 weeks Clients recovering from a laryngectomy are unable to take nutrition orally for about 10 to 14 days. During this time the client will receive nutrition via intravenous fluids, enteral feedings through a nasogastric tube, or parenteral nutrition. Protein and fluids are not limited. The tracheostomy tube is not cleaned with soap and water

The nurse, instructing a client regarding hormones, would include which of the following in these instructions? 1. Hormones are nonspecific 2. Hormones release triggers a rapid response 3. Hormones do not influence other hormones 4. The nervous system and hormones work together to maintain homeostasis

4. The nervous system and hormones work together to maintain homeostasis A close relationship between the endocrine and nervous systems is required to allow them to control homeostasis. The short-term rapid responses by the nervous system are balanced by the long-term responses from the endocrine system. Hormones are specific and can influence other hormones. This is what the nurse should instruct the client. The other choices are incorrect and should not be included in instructions to the client.

A client is hospitalized with an ongoing fever. The nurse learns that the client has had a recent infection. Currently the client is restless, diaphoretic, and agitated with the following vital signs: temperature 106F, pulse 114, blood pressure 180/80 mmHg. Which of the following disorders is the client most likely experiencing? 1. Addisonian crisis 2. Goiter 3. Myxedema 4. Thyroid crisis

4. Thyroid crisis Thyroid crisis is a serious form of hyperthyroidism that is life threatening. It is most likely to occur in persons who have been inadequately treated or undiagnosed. Infection, stress or emotional trauma, pregnancy, and medications may precipitate the event. Myxedema and Addisonian crisis would not produce a severe increase in blood pressure. Goiter tends to interfere with swallowing and breathing.

A client has been diagnosed with stage IV cancer of the larynx. The nurse realizes that which of the following surgeries is recommended for this type of cancer? 1. Hemilaryngectomy 2. Partial laryngectomy 3. Supraglottic laryngectomy 4. Total laryngectomy

4. Total laryngectomy In clients diagnosed with invasive or infiltrating tumors such as those of stage III or stage IV, the entire larynx is removed. The other surgeries only remove portions of the larynx and would be appropriate for lesser stages of the disease

A client is experiencing a gradual increase of pleuritic pain. In which of the following pulmonary conditions would the nurse expect to see this type of pain? 1. Pneumococcal pneumonia 2. Pneumothorax 3. Pulmonary embolism 4. Tuberculosis

4. Tuberculosis A more gradual onset of pleuritic pain is seen in tuberculosis and malignancy. Acute pleuritic pain is associated with pneumococcal pneumonia, pneumothorax, and pulmonary embolism.

A client is complaining of severe abdomen pain. The nurse realizes this client is experiencing which type of pain? 1. Neuralgia 2. Pathological 3. Somatic 4. Visceral

4. Visceral Visceral pain is pain arising from the body organs or gastrointestinal tract. Somatic pain is pain that originates from the bone, joints, muscles, skin, or connective pain. Neuralgia and pathological pain are both types of pain that result from injury to a nerve or malfunction of the neuronal transmission process or due to impaired regulation.

The nurse is providing instructions to a client receiving treatment for Cushings syndrome. Which of the following instructions would not be appropriate for this client? 1. Monitor glucose levels 2. Implement safety precautions 3. Wear medical identification 4. Volunteer at the hospital to prevent depression

4. Volunteer at the hospital to prevent depression A client diagnosed with Cushings syndrome is predisposed to falls, injury, and increased glucose levels. The client should wear an identification bracelet indicating her disease process. The client should avoid crowds and persons with infections.

A client diagnosed with chronic obstructive pulmonary disease has the complication of cor pulmonale. Which of the following instructions will be included in the clients discharge teaching? 1. Adjust oxygen higher depending on activity level 2. Increase sodium in the diet 3. Maintain bed rest 4. Weigh self daily, and call the physician with a weight gain of 2 pounds

4. Weigh self daily, and call the physician with a weight gain of 2 pounds A weight gain of greater than 2 pounds would indicate fluid retention and need to be reported to the physician. Oxygen would not be increased past the prescribed level because this may eliminate the clients drive to breathe. Increasing sodium will encourage fluid retention. Moderate activity is desired to maintain a level of cardiovascular health

The nurse is ensuring that a client is able to make knowledgeable decisions regarding an upcoming surgery and can provide informed consent. What is the responsibility of the nurse regarding informed consent? 1. Explain the surgical options 2. Explain the operative risks 3. Describe the operative procedure to be done 4. Witness a patients signature

4. Witness a patient's signature The nurse may concurrently sign that he has witnessed a patients signature. It is the physicians responsibility to explain the other answer choices.

A client, diagnosed with acute appendicitis, is experiencing abdominal pain. The best way for the nurse to describe this clients pain would be: 1. chronic 2. neuropathic 3. referred 4. acute

4. acute Acute pain onset is sudden and of short duration. Chronic pain is a sudden or slow onset of mild to severe pain that lasts longer than 6 months. Referred pain is the result of the transfer of visceral pain sensations to a body surface at a distance from the actual origin. Neuropathic pain is paroxysmal pain that occurs along the branches of a nerve.

A client is diagnosed with a large pneumothorax. The percussion note the nurse would expect to find is: 1. dullness 2. flatness 3. resonant 4. tympany

4. tympany Air-filled areas have a percussion note of tympany. A resonant note can be elicited by percussing a patient with normal lungs. Flatness is heard over bone and dullness is heard over the organs.


Related study sets

Biology - Chapter 7 - Cell Respiration

View Set